SlideShare ist ein Scribd-Unternehmen logo
1 von 182
INTEGRATED MEDICAL QUIZ-
THE FINALS
POTPOURRI
■ Total 12 question (2 questions per team)
■ Each time gets to keep the question for 1 min, after that the question bounces to next team
■ Pounce and bounce applicable
■ Pounce window – 30 secs
■ Team with the question and team with passed question will get 20 points for correct answer and no negative
■ Team with pounce will get 40 points for correct answer and minus 20 for wrong answer
■ Score will be announced by the score keeper at the end of this round
An 8 month old child had a seizure only hours after a meal.
When brought to the hospital, his blood sugar was found to
be 52 mg/dl. Physical examination showed enlargement of
liver, which was palpable 4 cm below margin. Further
examination revealed a blood pH of 7.29 with a high anion
gap and increase in uric acid.
He was started on uncooked cornstarch and showed
improvement.
Why was improvement seen on addition of cornstarch?
1
Uncooked cornstarch is a glucose polymer- a complex
carbohydrate.
Longterm treatment with uncooked cornstarch improves
growth in von-Gierke disease patients.
3 year old boy is referred for investigation of weight loss, failure to thrive and frequent diarrhoea.
Stools are semi-formed and greasy.
Child has pallor, is emaciated, is below 10th percentile for both height and weight.
He is irritable, listless and has markedly reduced appetite with severe muscle wasting at pelvic and
shoulder girdle.
Fecal fat excretion- 24hr fecal fat estimation is markedly increased.
Xylose excretion test- performed with 25g oral load of monosaccharide. Serum xylose is 80mg/dL
after one hour and urinary excretion is 1.4g in 5 hr period.
Hemoglobin-9.7g/dL
Serum protein and albumin slightly decreased
PBS shows hypochromic microcytic RBC.
Biopsy of jejenum showed infiltration of inflammatory cells in subepithelial layer and surface
epithelium cells were cuboidal.
How does presence of tissue transglutaminase causes accumulation of inflammatory cells ?
2
Tissue transglutaminase deamidates glutamine residues in
gluten to glutamic acid.
Deamidation produces a negative charge in gluten binding
them to HLA DQ-2 and DQ 8, causing stimulation of T cell
via APC.
Activated T cells secrete chemokine to recruit more T cells.
A 36 year old IV drug abuser, male, is a known case of AIDS since 2012. He presented
to emergency room with two-day history of fever, chills, cough, diarrhea and significant
weight loss. He has had poor complained with ART, not taking treatment since past 4
years. CD4 count is 354/mm³.
On examination, he was febrile (38.2°C) with a pulse of 114 beats/min and a blood
pressure of 124/73 mmHg. He had innumerable, widespread, macronodular skin lesions
that were heterogeneous in size, texture and colour. Multiple, diffuse 1 cm subcutaneous
masses, with and without associated dusky erythema of the overlying skin, were also
present, particularly over the chest wall.
The right wrist was swollen and tender along the ulnar border, with an overlying dusky
erythema of the skin. There was striking bilateral epitrochlear lymphadenopathy, most
prominent on the right side.
A biopsy of a skin lesion revealed the presence of faintly staining Gram-negative bacilli.
Which stain will help you to further establish the diagnosis?
3
Patient has bacillary angiomatosis - cutaneous and osseous
forms.
Warthin silver stain stains Bartonella henselae
A 58 year old male farmer presented to OPD with complaint
of shortness of breath, fatigue and spontaneous bleeding
from gums. On examination, patient had an enlarged spleen
and liver but no lymphadenopathy. Oral cavity examination
shows lesions corresponding to candida infection.
Blood examination revealed anaemia which is normocytic
normochromic type with monocytopenia
Immunophenotyping- CD19+, CD20+, CD11c+, CD25+
What mutation happens in a B cell which gives rise to
this pathology?
4
Patient has hairy cell leukaemia as evident by splenomegaly,
anaemia and bleeding manifestations with specific
immunophenotype.
BRAF V600E mutation takes place in late, activated memory
B cell
A 10 year old boy was admitted to a hospital with a 4 day history of high fever,
headache myalgia and arthralgia, and a 2 day history of melena and hemoglobinuria.
Patient lived in Kolkata and was visiting Delhi to meet relatives 4 days before he was
brought to the hospital. He developed chills and rigor during the journey.
On examination, conjunctival hyperemia and redness of tongue was noted along with
mild hepatosplenomegaly. Results from a blood test revealed the following-
White blood cells- 2500/mm³
Platelet count- 32000/mm³
Haematocrit level of 47.2%
Ultrasonography revealed ascites around liver.
What is the name of the vaccine now made available in 11 countries that can help
prevent this disease? What is the caution to be practised while its administration
to this child ?
5
Dengvaxia
Patient should be sero-posiitve before giving this vaccine.
An 8 year old boy came to a clinic, the doctor immediately noticed an abnormality in his gait
(left sided) where the stance phase of gait was abnormally shortened relative to the swing
phase.
Child was afebrile with stable vital signs and appeared to have a painful range of
movement at both hips and both knees.
ESR is slightly raised with normal CRP and normal blood counts.
He was referred to an orthopaedician. Initial X ray was normal.
MRI was done, following which doctor prescribed the following cast.
What is the diagnosis that the orthopaedician arrived at?
6
Perthes disease/ Osteonecrosis of hip
Patient presented with limp and hip pain
Initial radiographs are usually normal, MRI may show
decreased vascularity to femoral head.
A 35 year old female rice paddy field worker in Cameroon (tropical Africa) came to hospital
with a painful swelling of the conjunctiva of her left eye of ten days duration. Prior to the
swelling in eye, she had a crawling sensation near right side of nose followed by a swelling
in the nasal bridge. There was gradual increase in size of eye swelling with associated
pain, headache and mild fever.
On full body examination, three more nodules were seen, two on right shin and one on left
knee. The nodules were fully mobile.
Affected Eye examination revealed 6/60 visual acuity, normal intraocular pressure and
conjunctival congestion.
The cyst was excised and a live worm was extracted. The worm was white, slender and
elongated; about 4 cm in length and 0.5 mm in diameter and was covered with a cuticle
marked with multiple transverse striations.
Which bacteria is incriminated for acting as an endosymbiont in this worm’s sexual
development cycle?
7
Wolbachia is endosymbiont for Onchocerca volvulus
A 32 year old woman comes to the office due to persistent cough and shortness of
breath. She has had 3 episodes of pneumonia over the last 3 years. The patient
had severe sinusitis a year ago and an episode of bloody diarrhoea that required
hospitalisation 6 months ago. She usually responds to antibiotics but takes several
days to clear the infection. No other family members have similar problems. The
patient has not traveled outside the country. She does not use tobacco, alcohol or
illicit drugs and currently takes no medications. All of her immunizations are up to
date. Blood pressure is 130/80 mmHg and pulse is 90 bpm. BMI is 22kg/m2.
Physical exam reveals fine crackles over right lower lung field. No lower extremity
edema is present. Neck palpation doesn’t show any lymph node enlargement.
WBC count is 14,000/mm3. HIV testing is negative. CXR reveals a right lower lobe
infiltrate.
Apart from infections, what are the other conditions this patient is
susceptible to?
8
Patient has CVID - common variable immune deficiency
syndrome
Autoimmune conditions, particularly hemolytic anemia,
thrombocytopenia, and rheumatoid arthritis
At increased risk for malignancies, and have a particular
susceptibility to non-Hodgkin lymphomas (NHL)
A 63 year woman with chronic kidney disease is maintained
on daily peritoneal dialysis. Her past medical history is
notable for hypertension and A-fib. There is no history of
hypercoagulability in other family members. Her medications
include losartan and warfarin. 2 days ago, she noticed a
painful nodule on the trunk that progressed to necrosis and
skin ulceration. While discontinuation of a drug, the condition
subsequently subsided.
How did the drug cause this condition?
9
Patient has calciphylaxis due to CKD
for treatment, we remove any drugs which may contribute to
it; like vitamin D, warfarin, calcium supplements, iron
Vitamin K helps in regeneration of GLA1 matrix protein.
GLA1 matrix protein is important in preventing vascular
calcification. With warfarin, protein is not made and makes
patient susceptible.
A 27 year old woman is brought to emergency department by local paramedics. The patient
was found unconscious at the scene of a house fire. On examination, she does not appear to
have any burns. Black soot is present near patient’s nares and mouth. Her capillary refill time is
4 seconds. BP is 132/90 mmHg, heart rate is 122 bpm and regular, and respirations are
24/min.
100% Supplemental oxygen is administered. Lab results are as follows:
pH 7.15
PaO2 114 mmHg
PaCO2 33 mmHg
Bicarbonate 12 mEq/L
Lactic acid, venous 20 mg/dL
Which antidote should be administered to make this patient stable and reverse the
poisoning?
10
Patient is victim of house fire and has smoke inhalation
injury. Smoke contains multiple toxins absorbed systemically
- 2 most dangerous in closed spaces are HCN and CO.
Start hydroxocobalamin for HCN as exposure can be
inhalation, dermal or . HCN symptoms can develop within
minutes.
Patient was already started on O2 therapy for CO poisoning.
A 35-year-old male presents to the family physician for bilateral gynecomastia.
He observed a progressive increase in his breast size starting 6 months ago. He
is sexually active and denies any drug use.
Physical examination reveals bilateral gynecomastia and tenderness. The
genito-urinary examination shows a 1 cm nodule in the right testis.
Otherwise, the examination is within normal limits. The laboratory report shows:
LH 3 U/L, FSH 2 U/L, testosterone 270 ng/dL (Normal 3 -10 ng/dL), estradiol
115 pg/mL (Normal 20 - 60 pg/mL), beta HCG undetectable, AFP undetectable.
Explain the pathogenesis behind gynecomastia in this case.
11
Leydig cell tumors are the most common type of testicular
sex cord stromal tumors, which may occur in all age groups.
Leydig cells are the principal source of testosterone and are
capable of estrogen production, due to markedly
increased aromatase expression.
Endocrine manifestations in adults are gynecomastia.
A 56 year old woman was referred by her GP after she reported
seeing halos around lights through both eyes for few weeks.
She had no history of ocular trauma or prior refractive surgery.
Her best corrected visual acuity was 20/20 bilaterally
Patient is a known case of A-fib and has been taking anti-
arrhythmic and anti-coagulant drugs for the same for past 2 years.
What do you think this patient developed?
12
The patient developed amiodarone- induced vortex
keratopathy
Shows a whorl-like pattern of corneal epithelial deposits
bilaterally aka cornea verticillate
CONNECT
■ Total 12 question (2 questions per team)
■ Each time gets to keep the question for 1 min, after that the question bounces to next team
■ Pounce and bounce applicable
■ Pounce window – 30 secs
■ Team with the question and team with passed question will get 20 points for correct answer and no negative
■ Team with pounce will get 40 points for correct answer and minus 20 for wrong answer
■ Score will be announced by the score keeper at the end of this round
1
• REITAN TEST
• AGITATION
• GLUTAMATE AND ASTROCYTE SWELLING
• TIPSS WORSENS
HEPATIC ENCEPH
2
BIMODAL AGE
• MYASTHENIA GRAVIS
• OSTEOSARCOMA
• HODGKIN LYMPHOMA
• CROHN’S
3
CAUSES OF PERIPHERAL
NEUROPATHY
• UREMIA
• TANGIER
• AMYLOIDOSIS
• FABRY
4
FRAGILE X
• MACRORCHIDISM
• FRAGILE SITE
• LONG EARS AND CROWDED TEETH
• MVP
5
SRBCT
• NEUROBLASTOMA
• WILM’S
• MERKEL CELL
• EWING’S SARCOMA
6
BILIARY SLUDGE
• USG
• TPN
• CEFTRIAXONE is a CAUSE
• ASPIRATE SHOWS CRYSTALS WITH FACETS
7
IGG4 RELATED DISEASE
• MIKULICZ
• STORIFORM FIBROSIS
• RIEDEL THYRODITIS
• PSC- BEADED APPEARANCE
8
MS
• DAWSON FINGERS
• PAPILLITIS
• TEMPERATE CLIMATE
• UHTHOFF
9
SELENIUM
• 5’- DEIODINASE
• GLUTATHIONE PEROXIDASE
• PITYRIASIS VERSICOLOR
• THIOREDOXIN REDUCTASE
10
Infectious causes of cystic
lung disease
• Staphylococcal
• Coccidiomycosis
• Paragonimus westermanii
• Penumocystic jirovecii
11
Causes of Pseudotumor
cerebri
• GH
• Vitamin A
• Tetracycline
• OCP
12
Lead poisoning
• Basophilic stippling
• Lead paint
• Fanconi syndrome
• Lead lines
CROSSWORD
▪ Crossword puzzle consist of 8 across and 7 down
▪ Total- 15 questions
▪ Plus 10 points will be awarded for each correct answer
▪ No negative marking
▪ Score will be announced at the end of this round
Across
2. alpha 1 antitrypsin remnants are PAS+ and .......resistant
5. These cells are seen in alpha Iduronidase deficiency
6. Streptococcus associated with colon cancer
10. This process takes place in cell with microbes, caspase 1 being an important mediator and
effector
11. This is used for the treatment of AIP
13. Repeated attacks of venous thrombosis due to mucin secretion from a tumour
15. These granules contain ADP,ATP, calcium and serotonin
Down
1. Iron binding molecule with bactericidal property
3. Marker for mast cell activation
4. An ornithine decarboxylase inhibitor for reduction of excessive facial hair in females
7. Lack of lysosomal enzyme leading to storage of GM2 ganglioside in neurons
8. Replacement of cytoplasm of basophil cells of anterior pituitary in Cushing syndrome
9. Auscultatory finding in mechanical bowel obstruction
12. This virus productively infects only CD 36+ cells
14. Family of NAD dependent protein deacteylase functioning to adapt to stress to increase
longevity
CASE STUDY
■ This round 6 sub division – each having 4 set of question from a particular organ system (CVS, endocrinology, )
■ The team with the highest total score will be given the chance to choose its subdivision, followed by the team
with 2nd highest score and so on.
■ Each team is supposed to answer each question within 1 minute of reading of the question by the quiz master
■ Scoring as follows:
• 1st question : 20 points
• 2nd question : 30 points
• 3rd question : 40 points
• 4th question : 50 points
• If a team answers all 4 questions correctly, 50 bonus points will be awarded
▪ Each question will have negative marking equal to its weightage
▪ Participants may opt to pass a question to avoid negative marking
▪ Wrongly answering a question doesn’t a team preclude to not answering following questions
▪ Details about any part points will be dealt with respective questions as they appear, by the quizmaster
ENDOCRINOLOGY
NEUROLOGY
CVS
INFECTIOUS DISEASE
REPRODUCTIVE
HEMATOLOGY
ENDOCRINOLOGY
ENDOCRINOLOGY Question 1
A 30-yr old woman comes to the physician complaining of a
significant increase in her daily urinary volume and intense
thirst. The symptoms started a few weeks ago, and she has
to get up frequently at night to urinate and drink fluids. She
has no other medical problems. The patient does not use
tobacco, alcohol, or illicit drugs and has no know drug
allergies. Her mother developed diabetes mellitus as an
adolescent. Temperature is 98 F, BP 110/70 mmHg, pulse
75 bpm and respirations are 15/min. Fasting serum labs are
as follows:
Sodium 146 mEq/L
Potassium 4.0 mEq/L
Bicarbonate 24 mEq/L
BUN 15 mg/dL
Creatinine 0.9 mg/dL
Glucose 90 mg/dL
Serum osmolality 300 mOsm/kg
Urine osmolality 126 mOsm/kg
After 6 hours of water deprivation, labs are as follows :
Sodium 153 mEq/L
Serum osmolality 320 mOsm/kg
Urine osmolality 132 mOsm/kg
One hour after subcutaneous administration of DDAVP, urine osmolality is 395 mOsm/kg.
What is the most appropriate treatment for this patient with the route of administration? [ no part
points]
The patient suffers from central DI and will be treated with Desmopressin
The response to water restriction and desmopressin helps establish the diagnosis:
A submaximal increase in urine osmolality in response to water deprivation (but usually to
≥300 mosmol/kg), with desmopressin resulting in a rise in urine osmolality of more than
100 percent in complete central DI and 15 to 50 percent in partial central DI.
Submaximal rise in urine osmolality in response to water restriction (but to well below 300
mosmol/kg), with desmopressin producing little or no elevation in urine osmolality in
complete nephrogenic DI, and a small (<45 percent) elevation in urine osmolality with
partial nephrogenic DI.
Primary polydipsia will be associated with a rise in urine osmolality, usually to above 500
mosmol/kg, and no response to desmopressin since endogenous release is intact.
ENDOCRINOLOGY Question 2
A 60 yr old man who emigrated from Iraq comes to the physician complaining of dizziness, fatigue, and weight loss. He has experienced daily
fevers and cough for the past 2 months. The patient doesn’t use tobacco, alcohol, or illicit drugs and takes no medications. His BP is 98/54 mmHg
while standing, pulse 105 bpm and regular, and respirations are 14/min and unlabored. SaO2 is 98% on room air. Labs are as follows:
Sodium 132mEq/L
Potassium 5.9 mEq/L
Chloride 102 mEq/L
Creatinine 0.8 mg/dL
Glucose 55 mg/dL
Hb 10g/dL
Platelets 430,000/mm3
TLC 4500/mm3
N — 46%
L — 45%
E — 9%
CXR shows a right upper-lobe cavitary lesion.
Which acid-base disturbance is expected in this patient?
• Patient has Tuberculous adrenalitis —> Addison’s
• TB Adrenal gland is commonly associated with active pulmonary TB/ genitourinary TB
• A deficiency in mineralocorticoid produces hyperkalemic, hyponatremic normal AG metabolic acidosis
1. Aldosterone increases Na reabsorption -> creates negative potential needed for secretion for H+
2. Aldosterone increases H+ secretion in collecting tubule
3. Directly and indirectly increases synthesis of ammonia
ENDOCRINOLOGY Question 3
A 58 year old woman with type 2 DM is evaluated by her primary care
provider for a tingling sensation in her hands and feet. She has had type
2 DM for 15 years with intermittently poor control. Her most recent
HbA1C was 7.9%. She is currently managed with insulin detemir 40 units
daily and metformin 1000mg daily. On neurolgic examination, there is
loss of DTR at ankles bilaterally. DTR are 2+ at knees, biceps and triceps.
Sensation is decreased to pinprick and light touch bilaterally to the ankle
and wrists. She also has difficulty ascertaining if the great toe is being
held in the up or down position when her eyes are closed. She finds it
difficult to sleep at night sometimes due to pain in her legs. She is
diagnosed with distal sensory polyneuropathy due to her diabetes.
Which of the following medications has been approved by FDA for
treatment of pain with diabetic neuropathy?
1. Amitryptline
2. Gabapentin
3. Venlafaxine
4. Carbamazepine
5. Nortryptiline
6. Duloxetine
7. Pregabalin
8. Lidocaine
9. Morphine
10.Imipramine
Duloxetine
Amitryptiline
Pregabalin
Venlafaxine
ENDOCRINOLOGY Question 4
A 49 yr old man comes to the office due to a 3-month history of fatigue. He also
reports diffuse joint pain, finger swelling, and difficulty gripping objects with his
right hand. Patient has poorly controlled hypertension despite being compliant
with medications, a low-salt diet, and a regular exercise. Temperature is 98 F, BP
146/98 mm Hg, pulse 90/min, and respirations are 14/min. On examination, facial
features appear coarse and differ significantly from his driver’s license photo taken
3 years ago. The palms are sweaty and have doughy feel. Skin is oily. Multiple skin
tags are noted, particularly on the neck area. Tapping the ventral aspect of right
wrist produces shooting pain on lateral side of the right hand.
What should be the next step towards diagnosing this patient?
The patient suffers from Acromegaly
Most appropriate step is to measure IGF-1 levels.
Return to topics
NEUROLOGY
A 34 year old man comes to the emergency department at 2:00 am due to
several episodes of acute right periorbital pain for the past week. Pain wakes
him at night, spreading from behind his right eye to his face and right
temple, and lasts 30-45 minutes. He says that the pain is so severe that his
right eye waters and he has nasal drainage. The patient is restless and
continuously rocks back and forth or paces around the room. Physical
examination shows conjunctival injection and a constricted right pupil. Other
examination findings, including neurologic examination, are normal.
Which is most likely to quick improve this patient’s current condition?
NEUROLOGY Question 1
Patient has cluster headache and 100 % O2 is most likely to
improve his condition
A 65 year old female is brought to her office by her concerned son due to increasing
confusion, loss of mobility and stiff limbs. She tends to cry out for no reason. She
often screams and sees a “ lion roaring in the backyard”. She often sees cats in her
room, even though her son doesn’t see any. She has significant memory loss. She
never had “joint problems” before. She was previously treated with haloperidol, but
this exaggerated her rigidity. She is a non-smoker. She has non-significant past
psychiatric history. In the office, she appears alert, but disoriented and quite agitated.
Her blood pressure is 136/72 mm Hg, pulse 98/min and respirations are 16/min.
Physical examination reveals impaired visuospatial abilities, increased tone, normal
reflexes, and coarse resting tremors in extremities. Her CBC, electrolytes, creatinine,
glucose, LFTs, TSH and B12 levels are within normal range. Serology for syphilis is
negative.
If a neuropathological examination was done for this patient, which 2
immunohistochemical stains would be used to clinch the diagnosis?
NEUROLOGY Question 2
The patient suffers from Lewy body dementia - fluctuating
cognitive impairment and bizarre, visual hallucinations.
Spontaneous motor features of parkinsonism or exaggeration with
drugs suggests that.
2 immunehistochemical stains used for Lewy body dementia are -
Ubiquitin and Alpha-synnuclein
A 3 year old boy is in NICU for management of prematurity. For the
past few days, he has had increased spontaneous movements,
decreased tone, seizures and rapidly increasing head circumference.
The boy was delivered vaginally at 30 weeks POG by multiparous
women with cervical incompetence, birth weigh was 1.3 kg. Prenatal
lab studies and USG were normal. Rupture of membranes occurred 3
hours prior to birth. Amniotic fluid was clear, and there was no
maternal fever. Neonate’s vital signs show intermittent bradycardia
and apnea. Examination shows a lethargic neonate with weak and
high-pitched cry, tense fontanels and generalized hypotonia. CBC
shows anemia. CRP is normal. Head USG is shown below:
NEUROLOGY Question 3
What is the location of pathology and
what makes this location vulnerable to this?
In preterm infants, the site of origin of bleeding is generally in small blood
vessels in the germinal matrix in subependymal and the subventricular zone,
located between the caudate nucleus and the thalamus at the level of the
foramen of Monro.
What makes it vulnerable?
• A deficient structural support system- paucity of pericytes,
immature basal lamina, and deficiency of tight junctions and glial
fibrillary acidic protein (GFAP) in the astrocyte end-feet
• Fragile capillary network drains into a well-developed deep venous
system which changes direction in a U-turn fashion as it empties
into the internal cerebral vein. This is prone to stasis —> increased
CVP —> GMH
A 62 year old man is brought to primary care physician due to
worsening insomnia, confusion and memory loss over past 3 weeks.
He also has muscle twitching and gait problems and this has caused
several falls. He has no headache, fever, or urinary problems. He
consumes 3-4 pints of beer occasionally , but doesn’t consume
tobacco. His pulse is 82bpm, BP 126/82 mmHg, temperature is 98.7
F. He is poorly groomed and disoriented to date and time. On
cognitive testing, he misses 3/3 delayed recall items and is unable to
draw a clock. Patient also demonstrates abnormal finger-pointing
test and prominent agnosia. Pertinent physical findings include
nystagmus and positive extensor plantar response bilaterally.
NEUROLOGY Question 4
Lab Studies are as follows:
Hematocrit 40%
WBC 5500 cells/ mm3
A non-contrast head CT is normal.
EEG was done and report is shown.
For definitive diagnosis, apart from gold standard brain biopsy, what else can also be demonstrated?
Rapidly progressive dementia with triphase periodic wave complex
on EEG suggests CJD
Return to topics
CARDIOLOGY
A 49 year old man is well known to your emergency department for frequent
visits for alcohol intoxication. As the on-call doctor, you are called to
evaluate him for admission tonight. On arrival, he is only mildly intoxicated
and able to provide a passable history. He claims that for past 3 months, he
has been increasingly short of breath with even minimal exertion and is
experiencing overwhelming fatigue. He has also been awakening at night
with extreme shortness of breath only alleviated by sitting on edge of his
bed. His ankles have been swollen. On examination, BP is 140/45mm Hg, HR
122 bpm. He has a bounding carotid pulse and elevated JVP. His PMI is
laterally displaced, and you can auscultate a low-pitched cardiac sound
directly following the S2. You note glossitis and find that he lacks sensation
to light touch below the mid-shins bilaterally. Basic lab studies show:
CARDIOLOGY Question 1
Albumin 3.2 g/dL
Creatinine 1.4 mg/dL
Sodium 134 mEq/L
Transthoracic echo reveals a LV enlargement with EF 70%
What would you like to administer this patient to make him improve
rapidly?
Patient has beri-beri
Administering thiamine will improve his condition dramatically
You are evaluating a 52 heard old patient who presented to the
emergency department for chest pain and elevated cardiac
enzymes. On auscultation, you can easily hear a superficial,
continuous murmur at the mid-sternal level that was never
documented before. Echocardiography is limited but able to
visualise all cardiac valves, which appear completely normal.
Patient undergoes a right heart catheterization. O2 saturation
values are tabulated:
CARDIOLOGY Question 2
SVC 58%
RA 60%
Mid-coronary sinus 91%
RV 70%
PA 70%
What abnormality is accounting for this
patient’s symptoms?
Patient has coronary Arterio-venous fistula —> MI
Continuous murmur at mid-sternal level on left side with step up
of SaO2 in coronary sinus is suggestive of this diagnosis
A 28 year old man presents to emergency department for dyspnea on
exertion. He had an orthotopic heart transplant for non-ischemic
cardiomyopathy 5 years ago and, in general, has done quite well except for
one CMV reactivation within the first year. He reports that for past 3
months, he has noticed that with decreasing amounts of exertion he has
been having limiting dyspnea. He is adamant that he is experiencing no chest
pain or pressure during these episodes. He has been perfectly compliant
with his regimen of tacrolimus, mycophenolate mofetil and low-dose
prednisone. Echocardiography reveals a normal LV function with normal LV
thickness. His resting ECG shows normal sinus rhythm at rate of 80bpm.
What is the most likely cause of his symptoms?
CARDIOLOGY Question 3
Patient has coronary artery disease.
CMV infection precipitated this disease
Silent MI in this case because of heart transplant which causes
denervation
A 66 year old man is admitted to hospital for progressive dyspnea
on exertion and fatigue. He has a past medical history of tobacco
abuse and is widely traveled, recently returning from a multi
country trip through South America. On presentation, his heart
rate is 104 bpm and irregularly irregular. Blood pressure is 96/76
mmHg. You note an elevated jugular venous pulsation and marked
lower extremity edema. Echocardiogram reveals a LVEF 55%, and
images are shown below.
CARDIOLOGY Question 4
To elucidate the etiology of heart failure,
which is the most appropriate diagnostic
test to perform next?
Patient has restrictive cardiomyopathy
Glittering in left ventricle wall suggests amyloidosis as the cause
To evaluate for amyloidosis further, Serum protein and SPEP are
minimum requirements.
Return to topics
INFECTIOUS DISEASE
A 40 year old man is admitted to hospital with 2-3 weeks of fever,
tender lymph nodes, and right upper quadrant abdominal pain. He
reports progressive weight loss and malaise for over a year. On
examination, he is found to be febrile and frail with temporal wasting
and oral thrush. Matted, tender anterior cervical lymph nodes <1cm
and tender hepatomegaly are noted. He is diagnosed with AIDS (
CD4+ lymphocyte count =20 and HIV RNA 650,000 copies/mL). Blood
cultures grow M avium. He is started on rifabutin and clarithromycin,
as well as dapsone for Pneumocystis prophylaxis, and discharged
home 2 weeks later after his fever subsides. He follows up with an
HIV provider 4 weeks later and is started on tenofovir, emtricitabine
and efavirenz.
INFECTIOUS DISEASE Question 1
2 weeks later, he returns to clinic with fevers, neck pain, and
abdominal pain. His temperature is 100.2 F, HR 110bpm, and BP
110/64 mmHg. SaO2 is normal. His cervical nodes are now 2 cm in
size and extremely tender, and one has fistulized to his skin and is
draining yellow pus and is AFB positive. His hepatomegaly is
pronounced and tender.
What is the likely explanation for his presentation?
IRIS - immune reconstitution inflammatory syndrome
A 45 yr old woman presents with an 8 week history of new-onset
headache that is persistent and daily. She describes the pain as a
diffuse ache and rates it 7/10. It has been worsening over time.
She had seen her primary doctor who reassured her that her
physical examination was normal, and he prescribed ibuprofen as
needed for pain. This treatment didn’t alleviate her symptoms.
Yesterday, she awoke with double vision and a facial droop. The
patient lives in Pennsylvania and hikes frequently. She doesn’t
recall any tick bites. She had intermittent joint pains over this last
month. She denies rash.
INFECTIOUS DISEASE Question 2
She presents to ED with for further evaluation. On physical
examination, the patient has a temperature of 99.4 F. Vital signs
are normal. Neurologic exam demonstrates a complete right facial
droop. The left eye fails to abduct when looking laterally, although
right eye has full range of motion.
Fill this table correctly with arrows compared to normal CSF values.
[no part points]
• Patient has chronic meningitis - symptoms ≥4 weeks
• H/o headache, cranial nerve palsy —> basal meningitis affecting
cranial nerve roots
• Patient is at risk of lyme disease because of her behaviour and
geography. And has early disseminated disease
• Patient’s don’t remember tick bites or rash appearance which
develops weeks ago
INFECTIOUS DISEASE Question 3
A 38 year old woman with a history of diabetes mellitus,
hypertension, and chronic renal insufficiency reports comes to
emergency department complaining of double vision for 1 day. She
has required chronic hemodialysis for 8 years and often misses
appointments, including 4 of her last 8 sessions. She also notes 12
hours of facial swelling and difficulty speaking. She appears to be in
moderate distress. Her vital signs are notable for temperature of
102.2 F, Bp of 155/95 mmHg, HR of 110 bpm, and respirations are
25 breaths/min. Her head examination demonstrates right sided
proptosis, facial edema and a facial palsy. Lab examination reveals:
WBC count of 15,000 cells/ mm3, serum glucose of 225 mg/dL, serum
creatinine of 6.3 mg/dL and HbA1C 9.7%.
ABG on room air is as follows: ph 7.24, pCO2 20mmHg, pO2 100 mmHg.
She is immediately brought to ICU and needle aspirate of retro-orbital mass is
performed. On-site cytopathology reveals the following picture.
How does ketoacidosis make this patient susceptible to this disease?
Ketoacidosis patients are prone to rhino-cerebral mucormycosis
1. Rhizopus have an enzyme ketone reductase, which allows them
to thrive in high, glucose conditions
2. Serum from individuals in DKA promotes growth
3. DKA patients have elevated Fe iron levels in their serum, which
supports Rhizopus growth at alkaline pH
While on a business trip to Hokkaido, Japan, a 42 year old man
presents to emergency department with severe abdominal pain.
He has no past medical or surgical history. He recalls no recent
history of abdominal discomfort, diarrhoea, melena, bright red
colour per rectum, nausea, or vomiting prior to this acute episode.
He ate sashimi (thin sliced raw fish with soy sauce) at a local
restaurant 3 hours prior to his meeting. On examination, he is in
terrible distress and has dry heaves. Temperature is 98.8 F, HR 128
bpm, BP 174/92 mmHg.
INFECTIOUS DISEASE Question 4
Examination is notable for an extremely tender abdomen with
guarding and rebound tenderness. Bowel sounds are present and
hyperactive. Rectal examination is normal, and guaiac test is
negative. Pelvic examination is unremarkable. WBC count is 6700
cells/ mm3, Hct 42%. A complete metabolic panel and lipase and
amylase levels are all within normal limits. CT shows no
abnormality.
What do you think is the causal organism?
ANISAKIASIS
nematode infection where human is accidental host.
Occurs hours to days after ingesting eggs that settled into muscles
of fish.
Presentation mimics acute abdomen.
Endoscopy is diagnostic and curative
Return to topics
REPRODUCTIVE
An 18 year old woman comes to clinic with primary amenorrhea,
sexual infantilism, and clitoromegaly. She has a history of ambiguous
external genitalia noted at birth. Reviewing her records, you see that
laparotomy performed at 17 months of age revealed normal internal
female genitalia and ovarian biopsy performed at that time revealed
normal appearing primordial follicles. Lab studies today reveal a
normal female karyotype and high testosterone and
androstenedione concenterations. Estradiol and estrone are
undetectable in the serum. Serum FSH and LH levels are high.
Which drug(s) acts on the same site as the site of enzyme deficiency
and inhibit it? Name any 2
REPRODUCTIVE Question 1
Anastrozole
Letrozole
Vorozole
Exemestane
A 22 year old G0P0 professional tennis player comes to physician with a 6 month
history of amenorrheoa. Menarche occurred at age 11 and her menstrual cycles were
regular until 6 months ago. She has an intense exercise schedule and eats a high-
protein/ low-fat diet. Patient has no headaches or problems with her vision. She has
not lost any weight recently. She takes no medications and doesn’t smoke or drink
alcohol. Her mother has hypertension that is controlled on medication. Patient’s BMI
is 20kg/m2. Pregnancy test is negative. LH and FSH values are low and serum prolactin
is normal. No menstrual bleeding occurs after a 10 day MDPA challenge.
Name 4 peptides (of non-reproductive origin/ not pertaining to reproductive physiology
directly) implicated in suppressing normal menstrual cycle in this disease.
REPRODUCTIVE Question 2
Leptin dec
Ghrelin inc
Neuropeptide Y inc
CRH inc
A 27 year old nulliparous woman comes to office for evaluation on
intermittent left pelvic pain for last 8 months. She has noticed that exercise
exacerbates discomfort. Patient is sexually active with her husband and
stopped taking oral contraceptives 2 years ago with intention of having
children. Her last menstrual period was 2 weeks ago. Patient’s menstrual
cycles are around 27 days with bleeding that lasts 4 days. She had
trichomoniasis when she was an adolescent. Her temperature is 98.9 F, and
BP is 120/72 mmHg. Physical exam reveals normal sized uterus and enlarged
left adnexa. USG shows homogenous cystic appearing mass on left ovary but
is otherwise normal.
Which malignancy is this disease relaed to?
REPRODUCTIVE Question 3
Development of endometroid and clear cell carcinoma
A 17 year old nulliparous girl comes to the office after a pelvic
ultrasound to evaluate left adnexal fullness found incidentally on
physical examination. Her LMP was 3 weeks ago. Menses occur
every 32 days and last 5-6 days with 2 days of heavy flow. Patient is
sexually active with a new partner and uses condoms for
contraception. She was previously on OCP but discontinued due to
severe headaches. Patient has no medical problems and has had
no surgeries. She doesn’t use tobacco, alcohol or illicit drugs. Her
older sister recently had IVF due to infertility from endometriosis.
USG shows an 8cm left ovarian cyst with calcification and
hyperechoic nodules.
REPRODUCTIVE Question 4
If this patient were to develop headache, agitation, altered mental
status, seizures and frequent episodes of rigidity; what would the
diagnosis be? How to confirm the diagnosis?
Anti NMDA receptor encephalitis associated with mature cystic
teratoma
Confirmed by IgG antibodies to NR1 subunit of NMDA receptor in
serum or CSF
Return to topics
HEMATOLOGY
A 10 year old boy admitted due to upper respiratory tract infection. He also feels
intermitting burning pain of hands and feet, especially in this summer.
Immunization status was up to date. His past medical history was unremarkable
with a normal birth history and was taking no medications. The platelet count at
the time of referral was 950,000 cells/mm3. The physical examination was normal
with the exception of mild splenomegaly. His white blood cell count was raised,
and some platelets were enlarged. Hb, ESR and C-reactive protein were all within
normal ranges.
Cytogentics showed normal 46, XY karyotype. No BCR/ABL chimaeric transcript
was demonstrated by reverse transcription polymerase chain reaction.
What is the common pathway involved by the mutations in various genes implicated
for this disease?
HEMATOLOGY Question 1
JAK- STAT pathway , essential thrombocytosis
A 13 year old Caucasian female was admitted to hospital for petechiae and bruising of
skin in absence of organomegaly. A CBC revealed severe thrombocytopenia (PC 9800
cells/mm3). Idiopathic thrombocytopenic purpura (ITP) was suspected and IVIG and
steroids were given with good clinical response. During follow up, it was seen that the
platelet count decreased during the tapering of the steroids. One month after suspension
of the steroid therapy, cutaneous petechial haemorrhages and mucosal bleeding
occurred. Peripheral blood count was performed and low platelet count was discovered (
PC 8900 cell/mm3) along with anaemia (Hb 10.7 gm%, leucopenia TLC 2400 cell/ mm3)
and neutropenia (ANC 900 cells/ mm3) appeared during the follow-up period.
Bone marrow aspiration ruled out lymphoproliferative disorder. DAT was positive and
anti-neutrophil antibodies were positive with normal evaluation of remaining immune
response.
What is your diagnosis?
HEMATOLOGY Question 2
Evans syndorme
Immunepancytopenia
A 44 yr old obese woman undergoes elective cholecystectomy for cholelithiasis.
Postoperatively, she does well and discharged after 3 days. Two days after
discharge, she develops altered mental status and fever and is brought to
emergency department by her family. She takes an anti-depressant but is
otherwise healthy. Her temperature is 103 F, pulse 127 bpm, BP 110/78 mmHg,
and her respirations are 15/min. Examination is notable for confusion and a
well-healed surgical incision. Routine chemistries are drawn and show normal
electrolytes, BUN of 80 mg/dL and creatinine of 2.5 mg/dL, WBC count of
17300 cells/mm3, Hct 30% and platelet count of 25000 cells/mm3.
PBS shows schistocytes and confirms low platelets without clumping.
What is the inciting event and pathogenesis of this disease?
HEMATOLOGY Question 3
Acquired TTP, ADAMTS13
A 51 year old woman of Japanese origin came to hospital for pain in right side of neck and multiple skin
lesions which she had for two months. She gave no history of blood transfusion or other parenteral
therapy. She had lived in Japan her whole life. Physical examination revealed lymphadenopathy in both
sides of neck, axilla and inguinal region. Multiple small popular skin eruptions were seen on chest,
abdomen and upper extremities. He had no complaint of fever, sweating or weight loss. Abdominal CT
scan showed Hepatosplenomegaly.
Peripheral blood smear is shown here
Immunophenotyping is typical of helper T cell phenotype along with strong positivity for CD25.
What is the protein responsible for development of this pathology?
HEMATOLOGY Question 4
ATLL by HTLV-1
Clover leaf cell
Tax protein
Return to topics
THANK YOU FOR PARTICIPATION

Weitere ähnliche Inhalte

Was ist angesagt?

Mediquiz mains July 2018
Mediquiz mains July 2018Mediquiz mains July 2018
Mediquiz mains July 2018SAABAIIMSBBSR
 
Curioso LHMC prelims
Curioso LHMC prelimsCurioso LHMC prelims
Curioso LHMC prelimsRitwik Mishra
 
AIIMS Medicine Quiz prelims
AIIMS Medicine Quiz prelimsAIIMS Medicine Quiz prelims
AIIMS Medicine Quiz prelimsUmang Arora
 
Medillectual juniors 2017 (Prelims and Mains)
Medillectual juniors 2017 (Prelims and Mains)Medillectual juniors 2017 (Prelims and Mains)
Medillectual juniors 2017 (Prelims and Mains)Quitzkrieg
 
Open mediquiz with answers
Open mediquiz with answersOpen mediquiz with answers
Open mediquiz with answersSAABAIIMSBBSR
 
UCMS: Prelim Medical Quiz2018
UCMS: Prelim Medical Quiz2018 UCMS: Prelim Medical Quiz2018
UCMS: Prelim Medical Quiz2018 Illuminous
 
WISSEN MEDIQUIZ 2018 PRELIMS
WISSEN MEDIQUIZ 2018 PRELIMSWISSEN MEDIQUIZ 2018 PRELIMS
WISSEN MEDIQUIZ 2018 PRELIMSSAABAIIMSBBSR
 
A General Quiz with a bit of Medical Flavour
A General Quiz with a bit of Medical FlavourA General Quiz with a bit of Medical Flavour
A General Quiz with a bit of Medical FlavourJim Jacob Roy
 
Mediquiz , a Medical trivia Quiz
Mediquiz  , a Medical trivia QuizMediquiz  , a Medical trivia Quiz
Mediquiz , a Medical trivia QuizJim Jacob Roy
 
Gen-Med Quiz at AIIMS
Gen-Med Quiz at AIIMSGen-Med Quiz at AIIMS
Gen-Med Quiz at AIIMSNishant Nihar
 
SCB Gen med quiz finals 2019
SCB Gen med quiz finals 2019SCB Gen med quiz finals 2019
SCB Gen med quiz finals 2019Puranjan Dev
 
Farzana's Medical Quiz
Farzana's Medical QuizFarzana's Medical Quiz
Farzana's Medical QuizVidyuth Anand
 

Was ist angesagt? (20)

Senior Medillectuals- Prelims
Senior Medillectuals- PrelimsSenior Medillectuals- Prelims
Senior Medillectuals- Prelims
 
Mediquiz mains July 2018
Mediquiz mains July 2018Mediquiz mains July 2018
Mediquiz mains July 2018
 
Curioso LHMC prelims
Curioso LHMC prelimsCurioso LHMC prelims
Curioso LHMC prelims
 
Senior Medillectuals Mains
Senior Medillectuals MainsSenior Medillectuals Mains
Senior Medillectuals Mains
 
AIIMS Medicine Quiz prelims
AIIMS Medicine Quiz prelimsAIIMS Medicine Quiz prelims
AIIMS Medicine Quiz prelims
 
Medillectual juniors 2017 (Prelims and Mains)
Medillectual juniors 2017 (Prelims and Mains)Medillectual juniors 2017 (Prelims and Mains)
Medillectual juniors 2017 (Prelims and Mains)
 
Open mediquiz with answers
Open mediquiz with answersOpen mediquiz with answers
Open mediquiz with answers
 
Senior Medillectuals Prelims
Senior Medillectuals Prelims Senior Medillectuals Prelims
Senior Medillectuals Prelims
 
Curioso LHMC Mains
Curioso LHMC MainsCurioso LHMC Mains
Curioso LHMC Mains
 
Junior Medillectuals- Mains
Junior Medillectuals- MainsJunior Medillectuals- Mains
Junior Medillectuals- Mains
 
UCMS: Prelim Medical Quiz2018
UCMS: Prelim Medical Quiz2018 UCMS: Prelim Medical Quiz2018
UCMS: Prelim Medical Quiz2018
 
WISSEN MEDIQUIZ 2018 PRELIMS
WISSEN MEDIQUIZ 2018 PRELIMSWISSEN MEDIQUIZ 2018 PRELIMS
WISSEN MEDIQUIZ 2018 PRELIMS
 
Premier Medillectuals :- Prelims
Premier Medillectuals :- PrelimsPremier Medillectuals :- Prelims
Premier Medillectuals :- Prelims
 
A General Quiz with a bit of Medical Flavour
A General Quiz with a bit of Medical FlavourA General Quiz with a bit of Medical Flavour
A General Quiz with a bit of Medical Flavour
 
Mediquiz , a Medical trivia Quiz
Mediquiz  , a Medical trivia QuizMediquiz  , a Medical trivia Quiz
Mediquiz , a Medical trivia Quiz
 
Illuminati 2018 Medical Trivia Quiz AFMC
Illuminati 2018 Medical Trivia Quiz AFMC Illuminati 2018 Medical Trivia Quiz AFMC
Illuminati 2018 Medical Trivia Quiz AFMC
 
Gen-Med Quiz at AIIMS
Gen-Med Quiz at AIIMSGen-Med Quiz at AIIMS
Gen-Med Quiz at AIIMS
 
SCB Gen med quiz finals 2019
SCB Gen med quiz finals 2019SCB Gen med quiz finals 2019
SCB Gen med quiz finals 2019
 
Farzana's Medical Quiz
Farzana's Medical QuizFarzana's Medical Quiz
Farzana's Medical Quiz
 
Medical Trivia Quiz
Medical Trivia QuizMedical Trivia Quiz
Medical Trivia Quiz
 

Ähnlich wie UCMS:Final Integrated medical quiz 2018

A Case presentation on fever cough with breathlessness
A Case presentation on fever cough with breathlessnessA Case presentation on fever cough with breathlessness
A Case presentation on fever cough with breathlessnessDr. Md. Suzon Islam
 
2 severe respiratory infections in the icu
2 severe respiratory infections in the icu2 severe respiratory infections in the icu
2 severe respiratory infections in the icuIslam Ibrahim
 
2 severe respiratory infections in the icu
2 severe respiratory infections in the icu2 severe respiratory infections in the icu
2 severe respiratory infections in the icuIslam Ibrahim
 
Clinical cases from infection diseases hospital part 4
Clinical cases from infection diseases hospital part 4Clinical cases from infection diseases hospital part 4
Clinical cases from infection diseases hospital part 4drandreyst-p
 
Case based discussion on Listeria monocytogenes
Case based discussion on Listeria monocytogenesCase based discussion on Listeria monocytogenes
Case based discussion on Listeria monocytogenesdrmunnasraj
 
Clinical cases from infection diseases hospital
Clinical cases from infection diseases hospitalClinical cases from infection diseases hospital
Clinical cases from infection diseases hospitaldrandreyst-p
 
Premier Medillectuals - Mains
Premier Medillectuals - MainsPremier Medillectuals - Mains
Premier Medillectuals - MainsDhananjay Bansal
 
A CHILD WITH INCOMPLETE KAWASAKI DISEASE (4).pptx
A CHILD WITH INCOMPLETE KAWASAKI DISEASE (4).pptxA CHILD WITH INCOMPLETE KAWASAKI DISEASE (4).pptx
A CHILD WITH INCOMPLETE KAWASAKI DISEASE (4).pptxDrPNatarajan2
 
Unusual Manifestations of Dengue Fever
Unusual Manifestations of Dengue FeverUnusual Manifestations of Dengue Fever
Unusual Manifestations of Dengue FeverApollo Hospitals
 
Mcqs & case discussion meningitis
Mcqs & case discussion meningitisMcqs & case discussion meningitis
Mcqs & case discussion meningitisDR. ANKUR KUMAR
 
Path Quiz 2019 round2
Path Quiz 2019 round2Path Quiz 2019 round2
Path Quiz 2019 round2Illuminous
 
Krok 2 - 2011 Question Paper (General Medicine)
Krok 2 - 2011 Question Paper (General Medicine)Krok 2 - 2011 Question Paper (General Medicine)
Krok 2 - 2011 Question Paper (General Medicine)Eneutron
 
Clinical ScenarioREASON FOR CONSULTATION Desaturation to 64.docx
Clinical ScenarioREASON FOR CONSULTATION Desaturation to 64.docxClinical ScenarioREASON FOR CONSULTATION Desaturation to 64.docx
Clinical ScenarioREASON FOR CONSULTATION Desaturation to 64.docxbartholomeocoombs
 
History taking a case based discussion
History taking a case based discussionHistory taking a case based discussion
History taking a case based discussionPritom Das
 
Infective endocarditis-Neonate
 Infective endocarditis-Neonate Infective endocarditis-Neonate
Infective endocarditis-NeonateChandan Gowda
 
Krok 2 - 2012 Question Paper (General Medicine)
Krok 2 - 2012 Question Paper (General Medicine)Krok 2 - 2012 Question Paper (General Medicine)
Krok 2 - 2012 Question Paper (General Medicine)Eneutron
 
Typical & atypical clinical presentations of COVID-19 in children
Typical & atypical clinical presentations of COVID-19 in childrenTypical & atypical clinical presentations of COVID-19 in children
Typical & atypical clinical presentations of COVID-19 in childrenMoosaAllawati1
 

Ähnlich wie UCMS:Final Integrated medical quiz 2018 (20)

Case discussion
Case discussionCase discussion
Case discussion
 
A Case presentation on fever cough with breathlessness
A Case presentation on fever cough with breathlessnessA Case presentation on fever cough with breathlessness
A Case presentation on fever cough with breathlessness
 
2 severe respiratory infections in the icu
2 severe respiratory infections in the icu2 severe respiratory infections in the icu
2 severe respiratory infections in the icu
 
2 severe respiratory infections in the icu
2 severe respiratory infections in the icu2 severe respiratory infections in the icu
2 severe respiratory infections in the icu
 
Clinical cases from infection diseases hospital part 4
Clinical cases from infection diseases hospital part 4Clinical cases from infection diseases hospital part 4
Clinical cases from infection diseases hospital part 4
 
Case based discussion on Listeria monocytogenes
Case based discussion on Listeria monocytogenesCase based discussion on Listeria monocytogenes
Case based discussion on Listeria monocytogenes
 
Clinical cases from infection diseases hospital
Clinical cases from infection diseases hospitalClinical cases from infection diseases hospital
Clinical cases from infection diseases hospital
 
Premier Medillectuals - Mains
Premier Medillectuals - MainsPremier Medillectuals - Mains
Premier Medillectuals - Mains
 
April 8, 09 Ppt.
April 8, 09 Ppt.April 8, 09 Ppt.
April 8, 09 Ppt.
 
A CHILD WITH INCOMPLETE KAWASAKI DISEASE (4).pptx
A CHILD WITH INCOMPLETE KAWASAKI DISEASE (4).pptxA CHILD WITH INCOMPLETE KAWASAKI DISEASE (4).pptx
A CHILD WITH INCOMPLETE KAWASAKI DISEASE (4).pptx
 
Unusual Manifestations of Dengue Fever
Unusual Manifestations of Dengue FeverUnusual Manifestations of Dengue Fever
Unusual Manifestations of Dengue Fever
 
Mcqs & case discussion meningitis
Mcqs & case discussion meningitisMcqs & case discussion meningitis
Mcqs & case discussion meningitis
 
Path Quiz 2019 round2
Path Quiz 2019 round2Path Quiz 2019 round2
Path Quiz 2019 round2
 
Krok 2 - 2011 Question Paper (General Medicine)
Krok 2 - 2011 Question Paper (General Medicine)Krok 2 - 2011 Question Paper (General Medicine)
Krok 2 - 2011 Question Paper (General Medicine)
 
Clinical ScenarioREASON FOR CONSULTATION Desaturation to 64.docx
Clinical ScenarioREASON FOR CONSULTATION Desaturation to 64.docxClinical ScenarioREASON FOR CONSULTATION Desaturation to 64.docx
Clinical ScenarioREASON FOR CONSULTATION Desaturation to 64.docx
 
History taking a case based discussion
History taking a case based discussionHistory taking a case based discussion
History taking a case based discussion
 
Case 17 5-2017
Case 17 5-2017Case 17 5-2017
Case 17 5-2017
 
Infective endocarditis-Neonate
 Infective endocarditis-Neonate Infective endocarditis-Neonate
Infective endocarditis-Neonate
 
Krok 2 - 2012 Question Paper (General Medicine)
Krok 2 - 2012 Question Paper (General Medicine)Krok 2 - 2012 Question Paper (General Medicine)
Krok 2 - 2012 Question Paper (General Medicine)
 
Typical & atypical clinical presentations of COVID-19 in children
Typical & atypical clinical presentations of COVID-19 in childrenTypical & atypical clinical presentations of COVID-19 in children
Typical & atypical clinical presentations of COVID-19 in children
 

Kürzlich hochgeladen

The Most Attractive Hyderabad Call Girls Kothapet 𖠋 6297143586 𖠋 Will You Mis...
The Most Attractive Hyderabad Call Girls Kothapet 𖠋 6297143586 𖠋 Will You Mis...The Most Attractive Hyderabad Call Girls Kothapet 𖠋 6297143586 𖠋 Will You Mis...
The Most Attractive Hyderabad Call Girls Kothapet 𖠋 6297143586 𖠋 Will You Mis...chandars293
 
Call Girls Tirupati Just Call 9907093804 Top Class Call Girl Service Available
Call Girls Tirupati Just Call 9907093804 Top Class Call Girl Service AvailableCall Girls Tirupati Just Call 9907093804 Top Class Call Girl Service Available
Call Girls Tirupati Just Call 9907093804 Top Class Call Girl Service AvailableDipal Arora
 
Top Quality Call Girl Service Kalyanpur 6378878445 Available Call Girls Any Time
Top Quality Call Girl Service Kalyanpur 6378878445 Available Call Girls Any TimeTop Quality Call Girl Service Kalyanpur 6378878445 Available Call Girls Any Time
Top Quality Call Girl Service Kalyanpur 6378878445 Available Call Girls Any TimeCall Girls Delhi
 
Book Paid Powai Call Girls Mumbai 𖠋 9930245274 𖠋Low Budget Full Independent H...
Book Paid Powai Call Girls Mumbai 𖠋 9930245274 𖠋Low Budget Full Independent H...Book Paid Powai Call Girls Mumbai 𖠋 9930245274 𖠋Low Budget Full Independent H...
Book Paid Powai Call Girls Mumbai 𖠋 9930245274 𖠋Low Budget Full Independent H...Call Girls in Nagpur High Profile
 
Premium Call Girls Cottonpet Whatsapp 7001035870 Independent Escort Service
Premium Call Girls Cottonpet Whatsapp 7001035870 Independent Escort ServicePremium Call Girls Cottonpet Whatsapp 7001035870 Independent Escort Service
Premium Call Girls Cottonpet Whatsapp 7001035870 Independent Escort Servicevidya singh
 
(Low Rate RASHMI ) Rate Of Call Girls Jaipur ❣ 8445551418 ❣ Elite Models & Ce...
(Low Rate RASHMI ) Rate Of Call Girls Jaipur ❣ 8445551418 ❣ Elite Models & Ce...(Low Rate RASHMI ) Rate Of Call Girls Jaipur ❣ 8445551418 ❣ Elite Models & Ce...
(Low Rate RASHMI ) Rate Of Call Girls Jaipur ❣ 8445551418 ❣ Elite Models & Ce...parulsinha
 
Call Girls Kochi Just Call 9907093804 Top Class Call Girl Service Available
Call Girls Kochi Just Call 9907093804 Top Class Call Girl Service AvailableCall Girls Kochi Just Call 9907093804 Top Class Call Girl Service Available
Call Girls Kochi Just Call 9907093804 Top Class Call Girl Service AvailableDipal Arora
 
Call Girls Dehradun Just Call 9907093804 Top Class Call Girl Service Available
Call Girls Dehradun Just Call 9907093804 Top Class Call Girl Service AvailableCall Girls Dehradun Just Call 9907093804 Top Class Call Girl Service Available
Call Girls Dehradun Just Call 9907093804 Top Class Call Girl Service AvailableDipal Arora
 
(👑VVIP ISHAAN ) Russian Call Girls Service Navi Mumbai🖕9920874524🖕Independent...
(👑VVIP ISHAAN ) Russian Call Girls Service Navi Mumbai🖕9920874524🖕Independent...(👑VVIP ISHAAN ) Russian Call Girls Service Navi Mumbai🖕9920874524🖕Independent...
(👑VVIP ISHAAN ) Russian Call Girls Service Navi Mumbai🖕9920874524🖕Independent...Taniya Sharma
 
Call Girls Horamavu WhatsApp Number 7001035870 Meeting With Bangalore Escorts
Call Girls Horamavu WhatsApp Number 7001035870 Meeting With Bangalore EscortsCall Girls Horamavu WhatsApp Number 7001035870 Meeting With Bangalore Escorts
Call Girls Horamavu WhatsApp Number 7001035870 Meeting With Bangalore Escortsvidya singh
 
Top Rated Bangalore Call Girls Mg Road ⟟ 9332606886 ⟟ Call Me For Genuine S...
Top Rated Bangalore Call Girls Mg Road ⟟   9332606886 ⟟ Call Me For Genuine S...Top Rated Bangalore Call Girls Mg Road ⟟   9332606886 ⟟ Call Me For Genuine S...
Top Rated Bangalore Call Girls Mg Road ⟟ 9332606886 ⟟ Call Me For Genuine S...narwatsonia7
 
College Call Girls in Haridwar 9667172968 Short 4000 Night 10000 Best call gi...
College Call Girls in Haridwar 9667172968 Short 4000 Night 10000 Best call gi...College Call Girls in Haridwar 9667172968 Short 4000 Night 10000 Best call gi...
College Call Girls in Haridwar 9667172968 Short 4000 Night 10000 Best call gi...perfect solution
 
VIP Call Girls Indore Kirti 💚😋 9256729539 🚀 Indore Escorts
VIP Call Girls Indore Kirti 💚😋  9256729539 🚀 Indore EscortsVIP Call Girls Indore Kirti 💚😋  9256729539 🚀 Indore Escorts
VIP Call Girls Indore Kirti 💚😋 9256729539 🚀 Indore Escortsaditipandeya
 
Call Girls Varanasi Just Call 9907093804 Top Class Call Girl Service Available
Call Girls Varanasi Just Call 9907093804 Top Class Call Girl Service AvailableCall Girls Varanasi Just Call 9907093804 Top Class Call Girl Service Available
Call Girls Varanasi Just Call 9907093804 Top Class Call Girl Service AvailableDipal Arora
 
Call Girls Ludhiana Just Call 9907093804 Top Class Call Girl Service Available
Call Girls Ludhiana Just Call 9907093804 Top Class Call Girl Service AvailableCall Girls Ludhiana Just Call 9907093804 Top Class Call Girl Service Available
Call Girls Ludhiana Just Call 9907093804 Top Class Call Girl Service AvailableDipal Arora
 
VIP Hyderabad Call Girls Bahadurpally 7877925207 ₹5000 To 25K With AC Room 💚😋
VIP Hyderabad Call Girls Bahadurpally 7877925207 ₹5000 To 25K With AC Room 💚😋VIP Hyderabad Call Girls Bahadurpally 7877925207 ₹5000 To 25K With AC Room 💚😋
VIP Hyderabad Call Girls Bahadurpally 7877925207 ₹5000 To 25K With AC Room 💚😋TANUJA PANDEY
 
Bangalore Call Girls Nelamangala Number 9332606886 Meetin With Bangalore Esc...
Bangalore Call Girls Nelamangala Number 9332606886  Meetin With Bangalore Esc...Bangalore Call Girls Nelamangala Number 9332606886  Meetin With Bangalore Esc...
Bangalore Call Girls Nelamangala Number 9332606886 Meetin With Bangalore Esc...narwatsonia7
 
Call Girls Jabalpur Just Call 9907093804 Top Class Call Girl Service Available
Call Girls Jabalpur Just Call 9907093804 Top Class Call Girl Service AvailableCall Girls Jabalpur Just Call 9907093804 Top Class Call Girl Service Available
Call Girls Jabalpur Just Call 9907093804 Top Class Call Girl Service AvailableDipal Arora
 
Night 7k to 12k Navi Mumbai Call Girl Photo 👉 BOOK NOW 9833363713 👈 ♀️ night ...
Night 7k to 12k Navi Mumbai Call Girl Photo 👉 BOOK NOW 9833363713 👈 ♀️ night ...Night 7k to 12k Navi Mumbai Call Girl Photo 👉 BOOK NOW 9833363713 👈 ♀️ night ...
Night 7k to 12k Navi Mumbai Call Girl Photo 👉 BOOK NOW 9833363713 👈 ♀️ night ...aartirawatdelhi
 
Call Girls Gwalior Just Call 9907093804 Top Class Call Girl Service Available
Call Girls Gwalior Just Call 9907093804 Top Class Call Girl Service AvailableCall Girls Gwalior Just Call 9907093804 Top Class Call Girl Service Available
Call Girls Gwalior Just Call 9907093804 Top Class Call Girl Service AvailableDipal Arora
 

Kürzlich hochgeladen (20)

The Most Attractive Hyderabad Call Girls Kothapet 𖠋 6297143586 𖠋 Will You Mis...
The Most Attractive Hyderabad Call Girls Kothapet 𖠋 6297143586 𖠋 Will You Mis...The Most Attractive Hyderabad Call Girls Kothapet 𖠋 6297143586 𖠋 Will You Mis...
The Most Attractive Hyderabad Call Girls Kothapet 𖠋 6297143586 𖠋 Will You Mis...
 
Call Girls Tirupati Just Call 9907093804 Top Class Call Girl Service Available
Call Girls Tirupati Just Call 9907093804 Top Class Call Girl Service AvailableCall Girls Tirupati Just Call 9907093804 Top Class Call Girl Service Available
Call Girls Tirupati Just Call 9907093804 Top Class Call Girl Service Available
 
Top Quality Call Girl Service Kalyanpur 6378878445 Available Call Girls Any Time
Top Quality Call Girl Service Kalyanpur 6378878445 Available Call Girls Any TimeTop Quality Call Girl Service Kalyanpur 6378878445 Available Call Girls Any Time
Top Quality Call Girl Service Kalyanpur 6378878445 Available Call Girls Any Time
 
Book Paid Powai Call Girls Mumbai 𖠋 9930245274 𖠋Low Budget Full Independent H...
Book Paid Powai Call Girls Mumbai 𖠋 9930245274 𖠋Low Budget Full Independent H...Book Paid Powai Call Girls Mumbai 𖠋 9930245274 𖠋Low Budget Full Independent H...
Book Paid Powai Call Girls Mumbai 𖠋 9930245274 𖠋Low Budget Full Independent H...
 
Premium Call Girls Cottonpet Whatsapp 7001035870 Independent Escort Service
Premium Call Girls Cottonpet Whatsapp 7001035870 Independent Escort ServicePremium Call Girls Cottonpet Whatsapp 7001035870 Independent Escort Service
Premium Call Girls Cottonpet Whatsapp 7001035870 Independent Escort Service
 
(Low Rate RASHMI ) Rate Of Call Girls Jaipur ❣ 8445551418 ❣ Elite Models & Ce...
(Low Rate RASHMI ) Rate Of Call Girls Jaipur ❣ 8445551418 ❣ Elite Models & Ce...(Low Rate RASHMI ) Rate Of Call Girls Jaipur ❣ 8445551418 ❣ Elite Models & Ce...
(Low Rate RASHMI ) Rate Of Call Girls Jaipur ❣ 8445551418 ❣ Elite Models & Ce...
 
Call Girls Kochi Just Call 9907093804 Top Class Call Girl Service Available
Call Girls Kochi Just Call 9907093804 Top Class Call Girl Service AvailableCall Girls Kochi Just Call 9907093804 Top Class Call Girl Service Available
Call Girls Kochi Just Call 9907093804 Top Class Call Girl Service Available
 
Call Girls Dehradun Just Call 9907093804 Top Class Call Girl Service Available
Call Girls Dehradun Just Call 9907093804 Top Class Call Girl Service AvailableCall Girls Dehradun Just Call 9907093804 Top Class Call Girl Service Available
Call Girls Dehradun Just Call 9907093804 Top Class Call Girl Service Available
 
(👑VVIP ISHAAN ) Russian Call Girls Service Navi Mumbai🖕9920874524🖕Independent...
(👑VVIP ISHAAN ) Russian Call Girls Service Navi Mumbai🖕9920874524🖕Independent...(👑VVIP ISHAAN ) Russian Call Girls Service Navi Mumbai🖕9920874524🖕Independent...
(👑VVIP ISHAAN ) Russian Call Girls Service Navi Mumbai🖕9920874524🖕Independent...
 
Call Girls Horamavu WhatsApp Number 7001035870 Meeting With Bangalore Escorts
Call Girls Horamavu WhatsApp Number 7001035870 Meeting With Bangalore EscortsCall Girls Horamavu WhatsApp Number 7001035870 Meeting With Bangalore Escorts
Call Girls Horamavu WhatsApp Number 7001035870 Meeting With Bangalore Escorts
 
Top Rated Bangalore Call Girls Mg Road ⟟ 9332606886 ⟟ Call Me For Genuine S...
Top Rated Bangalore Call Girls Mg Road ⟟   9332606886 ⟟ Call Me For Genuine S...Top Rated Bangalore Call Girls Mg Road ⟟   9332606886 ⟟ Call Me For Genuine S...
Top Rated Bangalore Call Girls Mg Road ⟟ 9332606886 ⟟ Call Me For Genuine S...
 
College Call Girls in Haridwar 9667172968 Short 4000 Night 10000 Best call gi...
College Call Girls in Haridwar 9667172968 Short 4000 Night 10000 Best call gi...College Call Girls in Haridwar 9667172968 Short 4000 Night 10000 Best call gi...
College Call Girls in Haridwar 9667172968 Short 4000 Night 10000 Best call gi...
 
VIP Call Girls Indore Kirti 💚😋 9256729539 🚀 Indore Escorts
VIP Call Girls Indore Kirti 💚😋  9256729539 🚀 Indore EscortsVIP Call Girls Indore Kirti 💚😋  9256729539 🚀 Indore Escorts
VIP Call Girls Indore Kirti 💚😋 9256729539 🚀 Indore Escorts
 
Call Girls Varanasi Just Call 9907093804 Top Class Call Girl Service Available
Call Girls Varanasi Just Call 9907093804 Top Class Call Girl Service AvailableCall Girls Varanasi Just Call 9907093804 Top Class Call Girl Service Available
Call Girls Varanasi Just Call 9907093804 Top Class Call Girl Service Available
 
Call Girls Ludhiana Just Call 9907093804 Top Class Call Girl Service Available
Call Girls Ludhiana Just Call 9907093804 Top Class Call Girl Service AvailableCall Girls Ludhiana Just Call 9907093804 Top Class Call Girl Service Available
Call Girls Ludhiana Just Call 9907093804 Top Class Call Girl Service Available
 
VIP Hyderabad Call Girls Bahadurpally 7877925207 ₹5000 To 25K With AC Room 💚😋
VIP Hyderabad Call Girls Bahadurpally 7877925207 ₹5000 To 25K With AC Room 💚😋VIP Hyderabad Call Girls Bahadurpally 7877925207 ₹5000 To 25K With AC Room 💚😋
VIP Hyderabad Call Girls Bahadurpally 7877925207 ₹5000 To 25K With AC Room 💚😋
 
Bangalore Call Girls Nelamangala Number 9332606886 Meetin With Bangalore Esc...
Bangalore Call Girls Nelamangala Number 9332606886  Meetin With Bangalore Esc...Bangalore Call Girls Nelamangala Number 9332606886  Meetin With Bangalore Esc...
Bangalore Call Girls Nelamangala Number 9332606886 Meetin With Bangalore Esc...
 
Call Girls Jabalpur Just Call 9907093804 Top Class Call Girl Service Available
Call Girls Jabalpur Just Call 9907093804 Top Class Call Girl Service AvailableCall Girls Jabalpur Just Call 9907093804 Top Class Call Girl Service Available
Call Girls Jabalpur Just Call 9907093804 Top Class Call Girl Service Available
 
Night 7k to 12k Navi Mumbai Call Girl Photo 👉 BOOK NOW 9833363713 👈 ♀️ night ...
Night 7k to 12k Navi Mumbai Call Girl Photo 👉 BOOK NOW 9833363713 👈 ♀️ night ...Night 7k to 12k Navi Mumbai Call Girl Photo 👉 BOOK NOW 9833363713 👈 ♀️ night ...
Night 7k to 12k Navi Mumbai Call Girl Photo 👉 BOOK NOW 9833363713 👈 ♀️ night ...
 
Call Girls Gwalior Just Call 9907093804 Top Class Call Girl Service Available
Call Girls Gwalior Just Call 9907093804 Top Class Call Girl Service AvailableCall Girls Gwalior Just Call 9907093804 Top Class Call Girl Service Available
Call Girls Gwalior Just Call 9907093804 Top Class Call Girl Service Available
 

UCMS:Final Integrated medical quiz 2018

  • 3. ■ Total 12 question (2 questions per team) ■ Each time gets to keep the question for 1 min, after that the question bounces to next team ■ Pounce and bounce applicable ■ Pounce window – 30 secs ■ Team with the question and team with passed question will get 20 points for correct answer and no negative ■ Team with pounce will get 40 points for correct answer and minus 20 for wrong answer ■ Score will be announced by the score keeper at the end of this round
  • 4. An 8 month old child had a seizure only hours after a meal. When brought to the hospital, his blood sugar was found to be 52 mg/dl. Physical examination showed enlargement of liver, which was palpable 4 cm below margin. Further examination revealed a blood pH of 7.29 with a high anion gap and increase in uric acid. He was started on uncooked cornstarch and showed improvement. Why was improvement seen on addition of cornstarch? 1
  • 5.
  • 6. Uncooked cornstarch is a glucose polymer- a complex carbohydrate. Longterm treatment with uncooked cornstarch improves growth in von-Gierke disease patients.
  • 7. 3 year old boy is referred for investigation of weight loss, failure to thrive and frequent diarrhoea. Stools are semi-formed and greasy. Child has pallor, is emaciated, is below 10th percentile for both height and weight. He is irritable, listless and has markedly reduced appetite with severe muscle wasting at pelvic and shoulder girdle. Fecal fat excretion- 24hr fecal fat estimation is markedly increased. Xylose excretion test- performed with 25g oral load of monosaccharide. Serum xylose is 80mg/dL after one hour and urinary excretion is 1.4g in 5 hr period. Hemoglobin-9.7g/dL Serum protein and albumin slightly decreased PBS shows hypochromic microcytic RBC. Biopsy of jejenum showed infiltration of inflammatory cells in subepithelial layer and surface epithelium cells were cuboidal. How does presence of tissue transglutaminase causes accumulation of inflammatory cells ? 2
  • 8.
  • 9. Tissue transglutaminase deamidates glutamine residues in gluten to glutamic acid. Deamidation produces a negative charge in gluten binding them to HLA DQ-2 and DQ 8, causing stimulation of T cell via APC. Activated T cells secrete chemokine to recruit more T cells.
  • 10. A 36 year old IV drug abuser, male, is a known case of AIDS since 2012. He presented to emergency room with two-day history of fever, chills, cough, diarrhea and significant weight loss. He has had poor complained with ART, not taking treatment since past 4 years. CD4 count is 354/mm³. On examination, he was febrile (38.2°C) with a pulse of 114 beats/min and a blood pressure of 124/73 mmHg. He had innumerable, widespread, macronodular skin lesions that were heterogeneous in size, texture and colour. Multiple, diffuse 1 cm subcutaneous masses, with and without associated dusky erythema of the overlying skin, were also present, particularly over the chest wall. The right wrist was swollen and tender along the ulnar border, with an overlying dusky erythema of the skin. There was striking bilateral epitrochlear lymphadenopathy, most prominent on the right side. A biopsy of a skin lesion revealed the presence of faintly staining Gram-negative bacilli. Which stain will help you to further establish the diagnosis? 3
  • 11.
  • 12. Patient has bacillary angiomatosis - cutaneous and osseous forms. Warthin silver stain stains Bartonella henselae
  • 13. A 58 year old male farmer presented to OPD with complaint of shortness of breath, fatigue and spontaneous bleeding from gums. On examination, patient had an enlarged spleen and liver but no lymphadenopathy. Oral cavity examination shows lesions corresponding to candida infection. Blood examination revealed anaemia which is normocytic normochromic type with monocytopenia Immunophenotyping- CD19+, CD20+, CD11c+, CD25+ What mutation happens in a B cell which gives rise to this pathology? 4
  • 14.
  • 15. Patient has hairy cell leukaemia as evident by splenomegaly, anaemia and bleeding manifestations with specific immunophenotype. BRAF V600E mutation takes place in late, activated memory B cell
  • 16. A 10 year old boy was admitted to a hospital with a 4 day history of high fever, headache myalgia and arthralgia, and a 2 day history of melena and hemoglobinuria. Patient lived in Kolkata and was visiting Delhi to meet relatives 4 days before he was brought to the hospital. He developed chills and rigor during the journey. On examination, conjunctival hyperemia and redness of tongue was noted along with mild hepatosplenomegaly. Results from a blood test revealed the following- White blood cells- 2500/mm³ Platelet count- 32000/mm³ Haematocrit level of 47.2% Ultrasonography revealed ascites around liver. What is the name of the vaccine now made available in 11 countries that can help prevent this disease? What is the caution to be practised while its administration to this child ? 5
  • 17.
  • 18. Dengvaxia Patient should be sero-posiitve before giving this vaccine.
  • 19. An 8 year old boy came to a clinic, the doctor immediately noticed an abnormality in his gait (left sided) where the stance phase of gait was abnormally shortened relative to the swing phase. Child was afebrile with stable vital signs and appeared to have a painful range of movement at both hips and both knees. ESR is slightly raised with normal CRP and normal blood counts. He was referred to an orthopaedician. Initial X ray was normal. MRI was done, following which doctor prescribed the following cast. What is the diagnosis that the orthopaedician arrived at? 6
  • 20.
  • 21. Perthes disease/ Osteonecrosis of hip Patient presented with limp and hip pain Initial radiographs are usually normal, MRI may show decreased vascularity to femoral head.
  • 22. A 35 year old female rice paddy field worker in Cameroon (tropical Africa) came to hospital with a painful swelling of the conjunctiva of her left eye of ten days duration. Prior to the swelling in eye, she had a crawling sensation near right side of nose followed by a swelling in the nasal bridge. There was gradual increase in size of eye swelling with associated pain, headache and mild fever. On full body examination, three more nodules were seen, two on right shin and one on left knee. The nodules were fully mobile. Affected Eye examination revealed 6/60 visual acuity, normal intraocular pressure and conjunctival congestion. The cyst was excised and a live worm was extracted. The worm was white, slender and elongated; about 4 cm in length and 0.5 mm in diameter and was covered with a cuticle marked with multiple transverse striations. Which bacteria is incriminated for acting as an endosymbiont in this worm’s sexual development cycle? 7
  • 23.
  • 24. Wolbachia is endosymbiont for Onchocerca volvulus
  • 25. A 32 year old woman comes to the office due to persistent cough and shortness of breath. She has had 3 episodes of pneumonia over the last 3 years. The patient had severe sinusitis a year ago and an episode of bloody diarrhoea that required hospitalisation 6 months ago. She usually responds to antibiotics but takes several days to clear the infection. No other family members have similar problems. The patient has not traveled outside the country. She does not use tobacco, alcohol or illicit drugs and currently takes no medications. All of her immunizations are up to date. Blood pressure is 130/80 mmHg and pulse is 90 bpm. BMI is 22kg/m2. Physical exam reveals fine crackles over right lower lung field. No lower extremity edema is present. Neck palpation doesn’t show any lymph node enlargement. WBC count is 14,000/mm3. HIV testing is negative. CXR reveals a right lower lobe infiltrate. Apart from infections, what are the other conditions this patient is susceptible to? 8
  • 26.
  • 27. Patient has CVID - common variable immune deficiency syndrome Autoimmune conditions, particularly hemolytic anemia, thrombocytopenia, and rheumatoid arthritis At increased risk for malignancies, and have a particular susceptibility to non-Hodgkin lymphomas (NHL)
  • 28. A 63 year woman with chronic kidney disease is maintained on daily peritoneal dialysis. Her past medical history is notable for hypertension and A-fib. There is no history of hypercoagulability in other family members. Her medications include losartan and warfarin. 2 days ago, she noticed a painful nodule on the trunk that progressed to necrosis and skin ulceration. While discontinuation of a drug, the condition subsequently subsided. How did the drug cause this condition? 9
  • 29.
  • 30. Patient has calciphylaxis due to CKD for treatment, we remove any drugs which may contribute to it; like vitamin D, warfarin, calcium supplements, iron Vitamin K helps in regeneration of GLA1 matrix protein. GLA1 matrix protein is important in preventing vascular calcification. With warfarin, protein is not made and makes patient susceptible.
  • 31. A 27 year old woman is brought to emergency department by local paramedics. The patient was found unconscious at the scene of a house fire. On examination, she does not appear to have any burns. Black soot is present near patient’s nares and mouth. Her capillary refill time is 4 seconds. BP is 132/90 mmHg, heart rate is 122 bpm and regular, and respirations are 24/min. 100% Supplemental oxygen is administered. Lab results are as follows: pH 7.15 PaO2 114 mmHg PaCO2 33 mmHg Bicarbonate 12 mEq/L Lactic acid, venous 20 mg/dL Which antidote should be administered to make this patient stable and reverse the poisoning? 10
  • 32.
  • 33. Patient is victim of house fire and has smoke inhalation injury. Smoke contains multiple toxins absorbed systemically - 2 most dangerous in closed spaces are HCN and CO. Start hydroxocobalamin for HCN as exposure can be inhalation, dermal or . HCN symptoms can develop within minutes. Patient was already started on O2 therapy for CO poisoning.
  • 34. A 35-year-old male presents to the family physician for bilateral gynecomastia. He observed a progressive increase in his breast size starting 6 months ago. He is sexually active and denies any drug use. Physical examination reveals bilateral gynecomastia and tenderness. The genito-urinary examination shows a 1 cm nodule in the right testis. Otherwise, the examination is within normal limits. The laboratory report shows: LH 3 U/L, FSH 2 U/L, testosterone 270 ng/dL (Normal 3 -10 ng/dL), estradiol 115 pg/mL (Normal 20 - 60 pg/mL), beta HCG undetectable, AFP undetectable. Explain the pathogenesis behind gynecomastia in this case. 11
  • 35.
  • 36. Leydig cell tumors are the most common type of testicular sex cord stromal tumors, which may occur in all age groups. Leydig cells are the principal source of testosterone and are capable of estrogen production, due to markedly increased aromatase expression. Endocrine manifestations in adults are gynecomastia.
  • 37. A 56 year old woman was referred by her GP after she reported seeing halos around lights through both eyes for few weeks. She had no history of ocular trauma or prior refractive surgery. Her best corrected visual acuity was 20/20 bilaterally Patient is a known case of A-fib and has been taking anti- arrhythmic and anti-coagulant drugs for the same for past 2 years. What do you think this patient developed? 12
  • 38.
  • 39. The patient developed amiodarone- induced vortex keratopathy Shows a whorl-like pattern of corneal epithelial deposits bilaterally aka cornea verticillate
  • 41. ■ Total 12 question (2 questions per team) ■ Each time gets to keep the question for 1 min, after that the question bounces to next team ■ Pounce and bounce applicable ■ Pounce window – 30 secs ■ Team with the question and team with passed question will get 20 points for correct answer and no negative ■ Team with pounce will get 40 points for correct answer and minus 20 for wrong answer ■ Score will be announced by the score keeper at the end of this round
  • 42. 1
  • 43.
  • 44. • REITAN TEST • AGITATION • GLUTAMATE AND ASTROCYTE SWELLING • TIPSS WORSENS HEPATIC ENCEPH
  • 45. 2
  • 46.
  • 47. BIMODAL AGE • MYASTHENIA GRAVIS • OSTEOSARCOMA • HODGKIN LYMPHOMA • CROHN’S
  • 48. 3
  • 49.
  • 50. CAUSES OF PERIPHERAL NEUROPATHY • UREMIA • TANGIER • AMYLOIDOSIS • FABRY
  • 51. 4
  • 52.
  • 53. FRAGILE X • MACRORCHIDISM • FRAGILE SITE • LONG EARS AND CROWDED TEETH • MVP
  • 54. 5
  • 55.
  • 56. SRBCT • NEUROBLASTOMA • WILM’S • MERKEL CELL • EWING’S SARCOMA
  • 57. 6
  • 58.
  • 59. BILIARY SLUDGE • USG • TPN • CEFTRIAXONE is a CAUSE • ASPIRATE SHOWS CRYSTALS WITH FACETS
  • 60. 7
  • 61.
  • 62. IGG4 RELATED DISEASE • MIKULICZ • STORIFORM FIBROSIS • RIEDEL THYRODITIS • PSC- BEADED APPEARANCE
  • 63. 8
  • 64.
  • 65. MS • DAWSON FINGERS • PAPILLITIS • TEMPERATE CLIMATE • UHTHOFF
  • 66. 9
  • 67.
  • 68. SELENIUM • 5’- DEIODINASE • GLUTATHIONE PEROXIDASE • PITYRIASIS VERSICOLOR • THIOREDOXIN REDUCTASE
  • 69. 10
  • 70.
  • 71. Infectious causes of cystic lung disease • Staphylococcal • Coccidiomycosis • Paragonimus westermanii • Penumocystic jirovecii
  • 72. 11
  • 73.
  • 74. Causes of Pseudotumor cerebri • GH • Vitamin A • Tetracycline • OCP
  • 75. 12
  • 76.
  • 77. Lead poisoning • Basophilic stippling • Lead paint • Fanconi syndrome • Lead lines
  • 79. ▪ Crossword puzzle consist of 8 across and 7 down ▪ Total- 15 questions ▪ Plus 10 points will be awarded for each correct answer ▪ No negative marking ▪ Score will be announced at the end of this round
  • 80.
  • 81. Across 2. alpha 1 antitrypsin remnants are PAS+ and .......resistant 5. These cells are seen in alpha Iduronidase deficiency 6. Streptococcus associated with colon cancer 10. This process takes place in cell with microbes, caspase 1 being an important mediator and effector 11. This is used for the treatment of AIP 13. Repeated attacks of venous thrombosis due to mucin secretion from a tumour 15. These granules contain ADP,ATP, calcium and serotonin
  • 82. Down 1. Iron binding molecule with bactericidal property 3. Marker for mast cell activation 4. An ornithine decarboxylase inhibitor for reduction of excessive facial hair in females 7. Lack of lysosomal enzyme leading to storage of GM2 ganglioside in neurons 8. Replacement of cytoplasm of basophil cells of anterior pituitary in Cushing syndrome 9. Auscultatory finding in mechanical bowel obstruction 12. This virus productively infects only CD 36+ cells 14. Family of NAD dependent protein deacteylase functioning to adapt to stress to increase longevity
  • 84. ■ This round 6 sub division – each having 4 set of question from a particular organ system (CVS, endocrinology, ) ■ The team with the highest total score will be given the chance to choose its subdivision, followed by the team with 2nd highest score and so on. ■ Each team is supposed to answer each question within 1 minute of reading of the question by the quiz master ■ Scoring as follows: • 1st question : 20 points • 2nd question : 30 points • 3rd question : 40 points • 4th question : 50 points • If a team answers all 4 questions correctly, 50 bonus points will be awarded ▪ Each question will have negative marking equal to its weightage ▪ Participants may opt to pass a question to avoid negative marking ▪ Wrongly answering a question doesn’t a team preclude to not answering following questions ▪ Details about any part points will be dealt with respective questions as they appear, by the quizmaster
  • 87. ENDOCRINOLOGY Question 1 A 30-yr old woman comes to the physician complaining of a significant increase in her daily urinary volume and intense thirst. The symptoms started a few weeks ago, and she has to get up frequently at night to urinate and drink fluids. She has no other medical problems. The patient does not use tobacco, alcohol, or illicit drugs and has no know drug allergies. Her mother developed diabetes mellitus as an adolescent. Temperature is 98 F, BP 110/70 mmHg, pulse 75 bpm and respirations are 15/min. Fasting serum labs are as follows:
  • 88. Sodium 146 mEq/L Potassium 4.0 mEq/L Bicarbonate 24 mEq/L BUN 15 mg/dL Creatinine 0.9 mg/dL Glucose 90 mg/dL Serum osmolality 300 mOsm/kg Urine osmolality 126 mOsm/kg After 6 hours of water deprivation, labs are as follows : Sodium 153 mEq/L Serum osmolality 320 mOsm/kg Urine osmolality 132 mOsm/kg One hour after subcutaneous administration of DDAVP, urine osmolality is 395 mOsm/kg. What is the most appropriate treatment for this patient with the route of administration? [ no part points]
  • 89.
  • 90. The patient suffers from central DI and will be treated with Desmopressin The response to water restriction and desmopressin helps establish the diagnosis: A submaximal increase in urine osmolality in response to water deprivation (but usually to ≥300 mosmol/kg), with desmopressin resulting in a rise in urine osmolality of more than 100 percent in complete central DI and 15 to 50 percent in partial central DI. Submaximal rise in urine osmolality in response to water restriction (but to well below 300 mosmol/kg), with desmopressin producing little or no elevation in urine osmolality in complete nephrogenic DI, and a small (<45 percent) elevation in urine osmolality with partial nephrogenic DI. Primary polydipsia will be associated with a rise in urine osmolality, usually to above 500 mosmol/kg, and no response to desmopressin since endogenous release is intact.
  • 91. ENDOCRINOLOGY Question 2 A 60 yr old man who emigrated from Iraq comes to the physician complaining of dizziness, fatigue, and weight loss. He has experienced daily fevers and cough for the past 2 months. The patient doesn’t use tobacco, alcohol, or illicit drugs and takes no medications. His BP is 98/54 mmHg while standing, pulse 105 bpm and regular, and respirations are 14/min and unlabored. SaO2 is 98% on room air. Labs are as follows: Sodium 132mEq/L Potassium 5.9 mEq/L Chloride 102 mEq/L Creatinine 0.8 mg/dL Glucose 55 mg/dL Hb 10g/dL Platelets 430,000/mm3 TLC 4500/mm3 N — 46% L — 45% E — 9% CXR shows a right upper-lobe cavitary lesion. Which acid-base disturbance is expected in this patient?
  • 92.
  • 93. • Patient has Tuberculous adrenalitis —> Addison’s • TB Adrenal gland is commonly associated with active pulmonary TB/ genitourinary TB • A deficiency in mineralocorticoid produces hyperkalemic, hyponatremic normal AG metabolic acidosis 1. Aldosterone increases Na reabsorption -> creates negative potential needed for secretion for H+ 2. Aldosterone increases H+ secretion in collecting tubule 3. Directly and indirectly increases synthesis of ammonia
  • 94. ENDOCRINOLOGY Question 3 A 58 year old woman with type 2 DM is evaluated by her primary care provider for a tingling sensation in her hands and feet. She has had type 2 DM for 15 years with intermittently poor control. Her most recent HbA1C was 7.9%. She is currently managed with insulin detemir 40 units daily and metformin 1000mg daily. On neurolgic examination, there is loss of DTR at ankles bilaterally. DTR are 2+ at knees, biceps and triceps. Sensation is decreased to pinprick and light touch bilaterally to the ankle and wrists. She also has difficulty ascertaining if the great toe is being held in the up or down position when her eyes are closed. She finds it difficult to sleep at night sometimes due to pain in her legs. She is diagnosed with distal sensory polyneuropathy due to her diabetes. Which of the following medications has been approved by FDA for treatment of pain with diabetic neuropathy?
  • 95. 1. Amitryptline 2. Gabapentin 3. Venlafaxine 4. Carbamazepine 5. Nortryptiline 6. Duloxetine 7. Pregabalin 8. Lidocaine 9. Morphine 10.Imipramine
  • 96.
  • 98. ENDOCRINOLOGY Question 4 A 49 yr old man comes to the office due to a 3-month history of fatigue. He also reports diffuse joint pain, finger swelling, and difficulty gripping objects with his right hand. Patient has poorly controlled hypertension despite being compliant with medications, a low-salt diet, and a regular exercise. Temperature is 98 F, BP 146/98 mm Hg, pulse 90/min, and respirations are 14/min. On examination, facial features appear coarse and differ significantly from his driver’s license photo taken 3 years ago. The palms are sweaty and have doughy feel. Skin is oily. Multiple skin tags are noted, particularly on the neck area. Tapping the ventral aspect of right wrist produces shooting pain on lateral side of the right hand. What should be the next step towards diagnosing this patient?
  • 99.
  • 100. The patient suffers from Acromegaly Most appropriate step is to measure IGF-1 levels.
  • 103. A 34 year old man comes to the emergency department at 2:00 am due to several episodes of acute right periorbital pain for the past week. Pain wakes him at night, spreading from behind his right eye to his face and right temple, and lasts 30-45 minutes. He says that the pain is so severe that his right eye waters and he has nasal drainage. The patient is restless and continuously rocks back and forth or paces around the room. Physical examination shows conjunctival injection and a constricted right pupil. Other examination findings, including neurologic examination, are normal. Which is most likely to quick improve this patient’s current condition? NEUROLOGY Question 1
  • 104.
  • 105. Patient has cluster headache and 100 % O2 is most likely to improve his condition
  • 106. A 65 year old female is brought to her office by her concerned son due to increasing confusion, loss of mobility and stiff limbs. She tends to cry out for no reason. She often screams and sees a “ lion roaring in the backyard”. She often sees cats in her room, even though her son doesn’t see any. She has significant memory loss. She never had “joint problems” before. She was previously treated with haloperidol, but this exaggerated her rigidity. She is a non-smoker. She has non-significant past psychiatric history. In the office, she appears alert, but disoriented and quite agitated. Her blood pressure is 136/72 mm Hg, pulse 98/min and respirations are 16/min. Physical examination reveals impaired visuospatial abilities, increased tone, normal reflexes, and coarse resting tremors in extremities. Her CBC, electrolytes, creatinine, glucose, LFTs, TSH and B12 levels are within normal range. Serology for syphilis is negative. If a neuropathological examination was done for this patient, which 2 immunohistochemical stains would be used to clinch the diagnosis? NEUROLOGY Question 2
  • 107.
  • 108. The patient suffers from Lewy body dementia - fluctuating cognitive impairment and bizarre, visual hallucinations. Spontaneous motor features of parkinsonism or exaggeration with drugs suggests that. 2 immunehistochemical stains used for Lewy body dementia are - Ubiquitin and Alpha-synnuclein
  • 109. A 3 year old boy is in NICU for management of prematurity. For the past few days, he has had increased spontaneous movements, decreased tone, seizures and rapidly increasing head circumference. The boy was delivered vaginally at 30 weeks POG by multiparous women with cervical incompetence, birth weigh was 1.3 kg. Prenatal lab studies and USG were normal. Rupture of membranes occurred 3 hours prior to birth. Amniotic fluid was clear, and there was no maternal fever. Neonate’s vital signs show intermittent bradycardia and apnea. Examination shows a lethargic neonate with weak and high-pitched cry, tense fontanels and generalized hypotonia. CBC shows anemia. CRP is normal. Head USG is shown below: NEUROLOGY Question 3
  • 110. What is the location of pathology and what makes this location vulnerable to this?
  • 111.
  • 112. In preterm infants, the site of origin of bleeding is generally in small blood vessels in the germinal matrix in subependymal and the subventricular zone, located between the caudate nucleus and the thalamus at the level of the foramen of Monro. What makes it vulnerable? • A deficient structural support system- paucity of pericytes, immature basal lamina, and deficiency of tight junctions and glial fibrillary acidic protein (GFAP) in the astrocyte end-feet • Fragile capillary network drains into a well-developed deep venous system which changes direction in a U-turn fashion as it empties into the internal cerebral vein. This is prone to stasis —> increased CVP —> GMH
  • 113. A 62 year old man is brought to primary care physician due to worsening insomnia, confusion and memory loss over past 3 weeks. He also has muscle twitching and gait problems and this has caused several falls. He has no headache, fever, or urinary problems. He consumes 3-4 pints of beer occasionally , but doesn’t consume tobacco. His pulse is 82bpm, BP 126/82 mmHg, temperature is 98.7 F. He is poorly groomed and disoriented to date and time. On cognitive testing, he misses 3/3 delayed recall items and is unable to draw a clock. Patient also demonstrates abnormal finger-pointing test and prominent agnosia. Pertinent physical findings include nystagmus and positive extensor plantar response bilaterally. NEUROLOGY Question 4
  • 114. Lab Studies are as follows: Hematocrit 40% WBC 5500 cells/ mm3 A non-contrast head CT is normal. EEG was done and report is shown. For definitive diagnosis, apart from gold standard brain biopsy, what else can also be demonstrated?
  • 115.
  • 116. Rapidly progressive dementia with triphase periodic wave complex on EEG suggests CJD
  • 119. A 49 year old man is well known to your emergency department for frequent visits for alcohol intoxication. As the on-call doctor, you are called to evaluate him for admission tonight. On arrival, he is only mildly intoxicated and able to provide a passable history. He claims that for past 3 months, he has been increasingly short of breath with even minimal exertion and is experiencing overwhelming fatigue. He has also been awakening at night with extreme shortness of breath only alleviated by sitting on edge of his bed. His ankles have been swollen. On examination, BP is 140/45mm Hg, HR 122 bpm. He has a bounding carotid pulse and elevated JVP. His PMI is laterally displaced, and you can auscultate a low-pitched cardiac sound directly following the S2. You note glossitis and find that he lacks sensation to light touch below the mid-shins bilaterally. Basic lab studies show: CARDIOLOGY Question 1
  • 120. Albumin 3.2 g/dL Creatinine 1.4 mg/dL Sodium 134 mEq/L Transthoracic echo reveals a LV enlargement with EF 70% What would you like to administer this patient to make him improve rapidly?
  • 121.
  • 122. Patient has beri-beri Administering thiamine will improve his condition dramatically
  • 123. You are evaluating a 52 heard old patient who presented to the emergency department for chest pain and elevated cardiac enzymes. On auscultation, you can easily hear a superficial, continuous murmur at the mid-sternal level that was never documented before. Echocardiography is limited but able to visualise all cardiac valves, which appear completely normal. Patient undergoes a right heart catheterization. O2 saturation values are tabulated: CARDIOLOGY Question 2
  • 124. SVC 58% RA 60% Mid-coronary sinus 91% RV 70% PA 70% What abnormality is accounting for this patient’s symptoms?
  • 125.
  • 126. Patient has coronary Arterio-venous fistula —> MI Continuous murmur at mid-sternal level on left side with step up of SaO2 in coronary sinus is suggestive of this diagnosis
  • 127. A 28 year old man presents to emergency department for dyspnea on exertion. He had an orthotopic heart transplant for non-ischemic cardiomyopathy 5 years ago and, in general, has done quite well except for one CMV reactivation within the first year. He reports that for past 3 months, he has noticed that with decreasing amounts of exertion he has been having limiting dyspnea. He is adamant that he is experiencing no chest pain or pressure during these episodes. He has been perfectly compliant with his regimen of tacrolimus, mycophenolate mofetil and low-dose prednisone. Echocardiography reveals a normal LV function with normal LV thickness. His resting ECG shows normal sinus rhythm at rate of 80bpm. What is the most likely cause of his symptoms? CARDIOLOGY Question 3
  • 128.
  • 129. Patient has coronary artery disease. CMV infection precipitated this disease Silent MI in this case because of heart transplant which causes denervation
  • 130. A 66 year old man is admitted to hospital for progressive dyspnea on exertion and fatigue. He has a past medical history of tobacco abuse and is widely traveled, recently returning from a multi country trip through South America. On presentation, his heart rate is 104 bpm and irregularly irregular. Blood pressure is 96/76 mmHg. You note an elevated jugular venous pulsation and marked lower extremity edema. Echocardiogram reveals a LVEF 55%, and images are shown below. CARDIOLOGY Question 4
  • 131. To elucidate the etiology of heart failure, which is the most appropriate diagnostic test to perform next?
  • 132.
  • 133. Patient has restrictive cardiomyopathy Glittering in left ventricle wall suggests amyloidosis as the cause To evaluate for amyloidosis further, Serum protein and SPEP are minimum requirements.
  • 136. A 40 year old man is admitted to hospital with 2-3 weeks of fever, tender lymph nodes, and right upper quadrant abdominal pain. He reports progressive weight loss and malaise for over a year. On examination, he is found to be febrile and frail with temporal wasting and oral thrush. Matted, tender anterior cervical lymph nodes <1cm and tender hepatomegaly are noted. He is diagnosed with AIDS ( CD4+ lymphocyte count =20 and HIV RNA 650,000 copies/mL). Blood cultures grow M avium. He is started on rifabutin and clarithromycin, as well as dapsone for Pneumocystis prophylaxis, and discharged home 2 weeks later after his fever subsides. He follows up with an HIV provider 4 weeks later and is started on tenofovir, emtricitabine and efavirenz. INFECTIOUS DISEASE Question 1
  • 137. 2 weeks later, he returns to clinic with fevers, neck pain, and abdominal pain. His temperature is 100.2 F, HR 110bpm, and BP 110/64 mmHg. SaO2 is normal. His cervical nodes are now 2 cm in size and extremely tender, and one has fistulized to his skin and is draining yellow pus and is AFB positive. His hepatomegaly is pronounced and tender. What is the likely explanation for his presentation?
  • 138.
  • 139. IRIS - immune reconstitution inflammatory syndrome
  • 140. A 45 yr old woman presents with an 8 week history of new-onset headache that is persistent and daily. She describes the pain as a diffuse ache and rates it 7/10. It has been worsening over time. She had seen her primary doctor who reassured her that her physical examination was normal, and he prescribed ibuprofen as needed for pain. This treatment didn’t alleviate her symptoms. Yesterday, she awoke with double vision and a facial droop. The patient lives in Pennsylvania and hikes frequently. She doesn’t recall any tick bites. She had intermittent joint pains over this last month. She denies rash. INFECTIOUS DISEASE Question 2
  • 141. She presents to ED with for further evaluation. On physical examination, the patient has a temperature of 99.4 F. Vital signs are normal. Neurologic exam demonstrates a complete right facial droop. The left eye fails to abduct when looking laterally, although right eye has full range of motion. Fill this table correctly with arrows compared to normal CSF values. [no part points]
  • 142.
  • 143. • Patient has chronic meningitis - symptoms ≥4 weeks • H/o headache, cranial nerve palsy —> basal meningitis affecting cranial nerve roots • Patient is at risk of lyme disease because of her behaviour and geography. And has early disseminated disease • Patient’s don’t remember tick bites or rash appearance which develops weeks ago
  • 144. INFECTIOUS DISEASE Question 3 A 38 year old woman with a history of diabetes mellitus, hypertension, and chronic renal insufficiency reports comes to emergency department complaining of double vision for 1 day. She has required chronic hemodialysis for 8 years and often misses appointments, including 4 of her last 8 sessions. She also notes 12 hours of facial swelling and difficulty speaking. She appears to be in moderate distress. Her vital signs are notable for temperature of 102.2 F, Bp of 155/95 mmHg, HR of 110 bpm, and respirations are 25 breaths/min. Her head examination demonstrates right sided proptosis, facial edema and a facial palsy. Lab examination reveals:
  • 145. WBC count of 15,000 cells/ mm3, serum glucose of 225 mg/dL, serum creatinine of 6.3 mg/dL and HbA1C 9.7%. ABG on room air is as follows: ph 7.24, pCO2 20mmHg, pO2 100 mmHg. She is immediately brought to ICU and needle aspirate of retro-orbital mass is performed. On-site cytopathology reveals the following picture. How does ketoacidosis make this patient susceptible to this disease?
  • 146.
  • 147. Ketoacidosis patients are prone to rhino-cerebral mucormycosis 1. Rhizopus have an enzyme ketone reductase, which allows them to thrive in high, glucose conditions 2. Serum from individuals in DKA promotes growth 3. DKA patients have elevated Fe iron levels in their serum, which supports Rhizopus growth at alkaline pH
  • 148. While on a business trip to Hokkaido, Japan, a 42 year old man presents to emergency department with severe abdominal pain. He has no past medical or surgical history. He recalls no recent history of abdominal discomfort, diarrhoea, melena, bright red colour per rectum, nausea, or vomiting prior to this acute episode. He ate sashimi (thin sliced raw fish with soy sauce) at a local restaurant 3 hours prior to his meeting. On examination, he is in terrible distress and has dry heaves. Temperature is 98.8 F, HR 128 bpm, BP 174/92 mmHg. INFECTIOUS DISEASE Question 4
  • 149. Examination is notable for an extremely tender abdomen with guarding and rebound tenderness. Bowel sounds are present and hyperactive. Rectal examination is normal, and guaiac test is negative. Pelvic examination is unremarkable. WBC count is 6700 cells/ mm3, Hct 42%. A complete metabolic panel and lipase and amylase levels are all within normal limits. CT shows no abnormality. What do you think is the causal organism?
  • 150.
  • 151. ANISAKIASIS nematode infection where human is accidental host. Occurs hours to days after ingesting eggs that settled into muscles of fish. Presentation mimics acute abdomen. Endoscopy is diagnostic and curative
  • 154. An 18 year old woman comes to clinic with primary amenorrhea, sexual infantilism, and clitoromegaly. She has a history of ambiguous external genitalia noted at birth. Reviewing her records, you see that laparotomy performed at 17 months of age revealed normal internal female genitalia and ovarian biopsy performed at that time revealed normal appearing primordial follicles. Lab studies today reveal a normal female karyotype and high testosterone and androstenedione concenterations. Estradiol and estrone are undetectable in the serum. Serum FSH and LH levels are high. Which drug(s) acts on the same site as the site of enzyme deficiency and inhibit it? Name any 2 REPRODUCTIVE Question 1
  • 155.
  • 157. A 22 year old G0P0 professional tennis player comes to physician with a 6 month history of amenorrheoa. Menarche occurred at age 11 and her menstrual cycles were regular until 6 months ago. She has an intense exercise schedule and eats a high- protein/ low-fat diet. Patient has no headaches or problems with her vision. She has not lost any weight recently. She takes no medications and doesn’t smoke or drink alcohol. Her mother has hypertension that is controlled on medication. Patient’s BMI is 20kg/m2. Pregnancy test is negative. LH and FSH values are low and serum prolactin is normal. No menstrual bleeding occurs after a 10 day MDPA challenge. Name 4 peptides (of non-reproductive origin/ not pertaining to reproductive physiology directly) implicated in suppressing normal menstrual cycle in this disease. REPRODUCTIVE Question 2
  • 158.
  • 160. A 27 year old nulliparous woman comes to office for evaluation on intermittent left pelvic pain for last 8 months. She has noticed that exercise exacerbates discomfort. Patient is sexually active with her husband and stopped taking oral contraceptives 2 years ago with intention of having children. Her last menstrual period was 2 weeks ago. Patient’s menstrual cycles are around 27 days with bleeding that lasts 4 days. She had trichomoniasis when she was an adolescent. Her temperature is 98.9 F, and BP is 120/72 mmHg. Physical exam reveals normal sized uterus and enlarged left adnexa. USG shows homogenous cystic appearing mass on left ovary but is otherwise normal. Which malignancy is this disease relaed to? REPRODUCTIVE Question 3
  • 161.
  • 162. Development of endometroid and clear cell carcinoma
  • 163. A 17 year old nulliparous girl comes to the office after a pelvic ultrasound to evaluate left adnexal fullness found incidentally on physical examination. Her LMP was 3 weeks ago. Menses occur every 32 days and last 5-6 days with 2 days of heavy flow. Patient is sexually active with a new partner and uses condoms for contraception. She was previously on OCP but discontinued due to severe headaches. Patient has no medical problems and has had no surgeries. She doesn’t use tobacco, alcohol or illicit drugs. Her older sister recently had IVF due to infertility from endometriosis. USG shows an 8cm left ovarian cyst with calcification and hyperechoic nodules. REPRODUCTIVE Question 4
  • 164. If this patient were to develop headache, agitation, altered mental status, seizures and frequent episodes of rigidity; what would the diagnosis be? How to confirm the diagnosis?
  • 165.
  • 166. Anti NMDA receptor encephalitis associated with mature cystic teratoma Confirmed by IgG antibodies to NR1 subunit of NMDA receptor in serum or CSF
  • 169. A 10 year old boy admitted due to upper respiratory tract infection. He also feels intermitting burning pain of hands and feet, especially in this summer. Immunization status was up to date. His past medical history was unremarkable with a normal birth history and was taking no medications. The platelet count at the time of referral was 950,000 cells/mm3. The physical examination was normal with the exception of mild splenomegaly. His white blood cell count was raised, and some platelets were enlarged. Hb, ESR and C-reactive protein were all within normal ranges. Cytogentics showed normal 46, XY karyotype. No BCR/ABL chimaeric transcript was demonstrated by reverse transcription polymerase chain reaction. What is the common pathway involved by the mutations in various genes implicated for this disease? HEMATOLOGY Question 1
  • 170.
  • 171. JAK- STAT pathway , essential thrombocytosis
  • 172. A 13 year old Caucasian female was admitted to hospital for petechiae and bruising of skin in absence of organomegaly. A CBC revealed severe thrombocytopenia (PC 9800 cells/mm3). Idiopathic thrombocytopenic purpura (ITP) was suspected and IVIG and steroids were given with good clinical response. During follow up, it was seen that the platelet count decreased during the tapering of the steroids. One month after suspension of the steroid therapy, cutaneous petechial haemorrhages and mucosal bleeding occurred. Peripheral blood count was performed and low platelet count was discovered ( PC 8900 cell/mm3) along with anaemia (Hb 10.7 gm%, leucopenia TLC 2400 cell/ mm3) and neutropenia (ANC 900 cells/ mm3) appeared during the follow-up period. Bone marrow aspiration ruled out lymphoproliferative disorder. DAT was positive and anti-neutrophil antibodies were positive with normal evaluation of remaining immune response. What is your diagnosis? HEMATOLOGY Question 2
  • 173.
  • 175. A 44 yr old obese woman undergoes elective cholecystectomy for cholelithiasis. Postoperatively, she does well and discharged after 3 days. Two days after discharge, she develops altered mental status and fever and is brought to emergency department by her family. She takes an anti-depressant but is otherwise healthy. Her temperature is 103 F, pulse 127 bpm, BP 110/78 mmHg, and her respirations are 15/min. Examination is notable for confusion and a well-healed surgical incision. Routine chemistries are drawn and show normal electrolytes, BUN of 80 mg/dL and creatinine of 2.5 mg/dL, WBC count of 17300 cells/mm3, Hct 30% and platelet count of 25000 cells/mm3. PBS shows schistocytes and confirms low platelets without clumping. What is the inciting event and pathogenesis of this disease? HEMATOLOGY Question 3
  • 176.
  • 178. A 51 year old woman of Japanese origin came to hospital for pain in right side of neck and multiple skin lesions which she had for two months. She gave no history of blood transfusion or other parenteral therapy. She had lived in Japan her whole life. Physical examination revealed lymphadenopathy in both sides of neck, axilla and inguinal region. Multiple small popular skin eruptions were seen on chest, abdomen and upper extremities. He had no complaint of fever, sweating or weight loss. Abdominal CT scan showed Hepatosplenomegaly. Peripheral blood smear is shown here Immunophenotyping is typical of helper T cell phenotype along with strong positivity for CD25. What is the protein responsible for development of this pathology? HEMATOLOGY Question 4
  • 179.
  • 180. ATLL by HTLV-1 Clover leaf cell Tax protein
  • 182. THANK YOU FOR PARTICIPATION